Re: [Fwd: Re: [obm-l] Equação]

2008-12-22 Por tôpico Fellipe Rossi
>
> Ralph, uma pequena correção na fatoração:
>

  (y - 3x + 2)(y - 2) = -1

  Daí vem:

  y - 2 = 1 e y - 3x + 2 = -1

  ou

  y - 2 = - 1 e y - 3x + 2 = 1


Solução única nos inteiros: (2, 3)

Abrs,

Rossi


>
>
>
>  Mensagem original   Assunto: Re: [obm-l] Equação  Data: Mon,
> 22 Dec 2008 11:46:44 -0300  De: Carlos Nehab 
>   Responder
> a: obm-l@mat.puc-rio.br  Para: obm-l@mat.puc-rio.br  Referências:
> <2b5739960812190126x1062ce40g583136fa2575d...@mail.gmail.com><2b5739960812190126x1062ce40g583136fa2575d...@mail.gmail.com>
> <654708.31006...@web31701.mail.mud.yahoo.com><654708.31006...@web31701.mail.mud.yahoo.com>
> <2d483c770812210902w64adeebbra9e8e1d3d9573...@mail.gmail.com><2d483c770812210902w64adeebbra9e8e1d3d9573...@mail.gmail.com>
>
>
> Oi, Ralph,
>
> y^2 - 3 = x(3y - 6) = 3x(y - 2)  implica que y^2 - 3 é divisível por 3 que
> implica que y é divisível por 3; além disso, como y é impar (pois se for
> para fica par = impar  :-) ...) a única solução é y = 3 (-3 não serve)...
>
> Abraços,
> Nehab
>
>
> Ralph Teixeira escreveu:
>
> Rearrumando as coisas e fatorando:
>
> (y-3x+2)(y-2)=1
>
> Entao y-2=1 ou y-2=-1... Uma delas nao presta, a outra presta, entao sim,
> esta equacao soh tem uma solução no inteiros.
>
> Abraço,
>Ralph
>
> 2008/12/21 Eder Albuquerque 
>
>>   Olá a todos,
>>
>> Alguém poderia me confirmar se a equação abaixo tem mais de uma solução
>> nos inteiros:
>>
>> y^2 - 3 = x(3y - 6)
>>
>>
>> Cheguei facilmente a uma solução, mas não sei se pára aí.
>>
>> Obrigado.
>>
>>
>>
>>
>>
>>  --
>> Veja quais são os assuntos do momento no Yahoo! + Buscados: Top 
>> 10-
>> Celebridades-
>> Música-
>> Esportes
>>
>
>  =
> Instruções para entrar na lista, sair da lista e usar a lista em
> http://www.mat.puc-rio.br/~obmlistas/obm-l.html=
> =
> Instruções para entrar na lista, sair da lista e usar a lista em
> http://www.mat.puc-rio.br/~obmlistas/obm-l.html=
>


Re: [obm-l] Equação

2008-12-21 Por tôpico Fellipe Rossi
acho que ja responderam esse e-mail

2008/12/21 Eder Albuquerque 

> Olá a todos,
>
> Alguém poderia me confirmar se a equação abaixo tem mais de uma solução nos
> inteiros:
>
> y^2 - 3 = x(3y - 6)
>
>
> Cheguei facilmente a uma solução, mas não sei se pára aí.
>
> Obrigado.
>
>
>
>
>
> --
> Veja quais são os assuntos do momento no Yahoo! + Buscados: Top 
> 10-
> Celebridades-
> Música-
> Esportes
>


Re: [obm-l] Re: [obm-l] RE: [obm-l] (UNB) TRÊS NÚMEROS

2008-12-21 Por tôpico Fellipe Rossi
Bouskela, o sistema está errado.
O correto:
P + Q + R = 75
P = 3.Q + 2
Q = 3.R + 2

O que dá: P = 53, Q = 17 e R = 5

Abrs,

Rossi


2008/12/21 grego 

> Olá!
> Faz sentido (P, Q, R) = (53, 17, 5)?
> Um abraço!
> Grego
>
>
> --- Em *dom, 21/12/08, Albert Bouskela * escreveu:
>
> De: Albert Bouskela 
> Assunto: [obm-l] RE: [obm-l] (UNB) TRÊS NÚMEROS
> Para: obm-l@mat.puc-rio.br
> Data: Domingo, 21 de Dezembro de 2008, 1:20
>
>
>  Olá!
>
> O sistema de equações fica assim:
>
>
>
> 1...   P+Q+R = 75
>
> 2...   P/3 = Q+2
>
> 3...   Q/3 = R+2
>
>
>
> A solução deste sistema é:
>
>
>
> P = 717/13, Q = 213/13, R = 45/13
>
>
>
> Logo, "P", "Q" e "R" não são inteiros – verifique o enunciado!
>
>
>
> *AB*
>
> bousk...@msn.com
>
>
>
> *From:* owner-ob...@mat.puc-rio.br [mailto:owner-ob...@mat.puc-rio.br] *On
> Behalf Of *arkon
> *Sent:* Sunday, December 21, 2008 12:43 AM
> *To:* obm-l@mat.puc-rio.br
> *Subject:* [obm-l] (UNB) TRÊS NÚMEROS
>
>
> Alguém pode resolver essa:
>
> Considere três números inteiros positivos P, Q e R tal que P+Q+R=75.
> Sabe-se que a divisão de P por 3 não é exata, tem quociente Q e o resto é o
> maior possível. A divisão de Q por 3  também não é exata, tem quociente R e
> o resto também é o maior possível. Nesse caso, P, Q e R são números primos?
>
> =
> Instru絥s para entrar na lista, sair da lista e usar a lista em
> http://www.mat.puc-rio.br/~obmlistas/obm-l.html=
>
>
> --
> Veja quais são os assuntos do momento no Yahoo! + Buscados: Top 
> 10-
> Celebridades-
> Música-
> Esportes
>


Re: [obm-l] probabilidade

2008-12-15 Por tôpico Fellipe Rossi
Dependerá da quantidade de alunos da turma

2008/12/15 Jefferson Franca 

> Ontem, recebi uma questão que ainda não resolvi, será que alguém pode me
> ajudar?
> Lá vai: Um aluno entrevistou sua turma para saber a intenção de votos numa
> pesquisa para representante dela e notou que houve um empate técnico, metade
> da turma votaria no candidato A, enquanto que a outra metade votaria no
> candidato B. Bem, um instituto de pesquisa escolheu aleatoriamente 4 alunos
> dessa turma, qual será a probabilidade desse instituto chegar à mesma
> conclusão que o aluno?
> Obrigado
>
> --- Em *sex, 31/10/08, Ralph Teixeira * escreveu:
>
> De: Ralph Teixeira 
> Assunto: Re: [obm-l] exercicio simples de probabilidade
> Para: obm-l@mat.puc-rio.br
> Data: Sexta-feira, 31 de Outubro de 2008, 23:58
>
>  Para mim, estao faltando dados... Agora, se voce me disser que:
>
> i) Em cada partida, a chance de A vencer eh p;
> ii) As partidas sao independentes entre si;
>
> Entao (ainda nao estah claro qual eh a pergunta, entao apresento duas
> respostas):
>
> Pr(A vencer exatamente 4 partidas) = C(6,4).p^4.(1-p)^2
> Pr(A vencer 4 ou mais partidas) = C(6,4).p^4.(1-p)^2+C(6,5).p^5.(1-p)+p^6
>
> Em particular, se p=50%, entao:
>
> Pr(A vencer exatamente 4) = 15/64 = 23.4375%
> Pr(A vencer pelo menos 4) = 11/32 = 34.375%
>
> Abraco,
> Ralph
>
> 2008/10/31 Graciliano Antonio Damazo 
>
>>   Caros amigos da lista, tenho uma questao simples de probabilidade que
>> resultou numa discussao na resolução da mesma numa aula de reforço que eu
>> estava estagiando la vai...mas não vale rirrsrs(brincadeira):
>>
>> 1) Dois times A e B jogam 6 partidas entre si. Qual a probabilidade do
>> time A vencer 4 dessas partidas?
>>
>> Gostaria de saber como vocês interpretam essa questão. Muito obrigado pela
>> atenção desde já.
>>
>>  --
>> Novos endereços, o Yahoo! que você conhece. Crie um email 
>> novocom
>>  a sua cara @
>> ymail.com ou @rocketmail.com.
>>
>
>
> --
> Veja quais são os assuntos do momento no Yahoo! + Buscados: Top 
> 10-
> Celebridades-
> Música-
> Esportes
>


Re: [obm-l] Contagem

2008-11-23 Por tôpico Fellipe Rossi
JOSE AIRTON.
Se você tem 10 amigos, de quantas formas você pode escolher 3 amigos pra
viajar com vc?


2008/11/23 JOSE AIRTON CARNEIRO <[EMAIL PROTECTED]>

> Perdão a todos da lista em especial ao João e ao graciliano, mas insisto
> porque tenho convicção que estou correto.
> Se tenho 10 números de 1 a 10, então posso escolher 3 desses números
> distintos de: A10,3 = 720. Ou seja
> {(1,2,3),(1,3,2),(2,4,6),(6,10,8),(10,9,8),(4,2,10)..}.
> O problema é que dessas 720 ternas, só nos interessa aquelas cuja soma é
> PAR.
> Dai a soma de 3 números é PAR quando: a) Os três forem pares (PAR - PAR -
> PAR)  ou b) 1 par e 2 ímpares (PAR - ÍMPAR - ÍMPAR).
> (PAR - PAR - PAR) = A5,3 = 60.
> (PAR - ÍMPAR - ÍMPAR) = A5,1 . A5,2 = 100.
> Logo temos 160 possibilidades de escolher 3 números distintos de 1 a 10 de
> modo que sua soma seja par.
>
>
>
>
> Em 23/11/08, João Luís <[EMAIL PROTECTED]> escreveu:
>>
>>  Olá José,
>>
>> Pois é, o problema não pede que se forme um número com os algarismos; na
>> verdade, nem se fala em "algarismos", e sim em "números de 1 a
>> 10". Inclusive, o próprio fato de o 10 estar incluído já mostra que não se
>> trata de formar números.
>>
>> Deve-se simplesmente escolher 3 números de 1 a 10 e verificar a paridade
>> da soma.
>>
>> Concorda?
>>
>> Um abraço a todos,
>>
>> João Luís.
>>
>>
>> - Original Message -
>>
>> *From:* JOSE AIRTON CARNEIRO <[EMAIL PROTECTED]>
>> *To:* obm-l@mat.puc-rio.br
>>  *Sent:* Sunday, November 23, 2008 2:43 PM
>> *Subject:* Re: [obm-l] Contagem
>>
>>
>> Olá João, posso até estar errado mas acho que é exatamente isso que o
>> problema pede.
>> Esse é nitidamente um problema de Arranjos.
>> Suponhamos que eu escolha 2 - 4 - 6 nessa ordem formando o nº 246 a soma
>> de seus algarismos é par.
>> E se eu escolher 4 - 6 - 2 nessa ordem formando o nº 462 também a soma de
>> seus algarismos é par.
>> São duas maneiras distintas de se escolher esses 3 nºs cuja soma é par. O
>> mesmo acontece com os PII.
>> Que argumento você usaria para descartar a escolha do 462?
>>
>>
>>
>>
>> Em 23/11/08, João Luís <[EMAIL PROTECTED]> escreveu:
>>>
>>>  Não é isso o que a questão pede
>>>
>>> - Original Message -
>>> *From:* Fellipe Rossi <[EMAIL PROTECTED]>
>>> *To:* obm-l@mat.puc-rio.br
>>> *Sent:* Saturday, November 22, 2008 6:21 PM
>>> *Subject:* Re: [obm-l] Contagem
>>>
>>>
>>> essa "escolha" tem que ser melhor definida.
>>>
>>> Por exemplo, se forem fichas numeradas em uma urna e retiram-se 3, um de
>>> cada vez, a ordem importa. Quer dizer, tirar 3-5-6 é uma retirada diferente
>>> de 5-3-6 não em relação aos números, mas em relação às fichas.
>>>
>>>
>>> Pensando, por exemplo, em probabilidade. A probabilidade de se retirar I
>>> I P, nessa ordem, é menor do que em uma ordem qualquer.
>>>
>>>
>>>
>>> Se qualquer forma, acho que o gabarito dessa questão é 60 realmente.
>>>
>>>
>>> []`s
>>>
>>> 2008/11/22 Walter Tadeu Nogueira da Silveira <[EMAIL PROTECTED]>
>>>
>>>> Concordo com o João
>>>>
>>>> Aliás, postei enganado o IPP. Queria por o IIP que a conta também dá 50.
>>>> O PPP dá 10. Pareceu a todos que a ordem não faria diferença.
>>>> A parte boa foi que apesar do gabarito oficial, nenhum aluno concordou.
>>>> Obrigado a todos!
>>>>
>>>>
>>>>  2008/11/22 João Luís <[EMAIL PROTECTED]>
>>>>
>>>>
>>>>>  Com dois pares e um ímpar, a soma dos três não será par.
>>>>>
>>>>> Para mim, a solução desse problema é a seguinte:
>>>>>
>>>>> Para que a soma dos três seja para, podemos escolher "nenhum ímpar e
>>>>> três pares" (10 modos) ou "dois ímpares e um par" (50 modos), não 
>>>>> importando
>>>>> a ordem da escolha, em virtude da comutatividade da adição.
>>>>>
>>>>> Portanto, teremos 60 escolhas.
>>>>>
>>>>> Um abraço a todos,
>>>>>
>>>>> João Luís.
>>>>>
>>>>>  - Original Message -
>>>>> *From:* Antonio Neto <[EMAIL PROTECTED]>
>>>>> *To:* obm-l@mat.puc-rio.br
>>>>>   *Sent:* Satu

Re: [obm-l] Contagem

2008-11-22 Por tôpico Fellipe Rossi
essa "escolha" tem que ser melhor definida.
Por exemplo, se forem fichas numeradas em uma urna e retiram-se 3, um de
cada vez, a ordem importa. Quer dizer, tirar 3-5-6 é uma retirada diferente
de 5-3-6 não em relação aos números, mas em relação às fichas.

Pensando, por exemplo, em probabilidade. A probabilidade de se retirar I I
P, nessa ordem, é menor do que em uma ordem qualquer.

Se qualquer forma, acho que o gabarito dessa questão é 60 realmente.

[]`s

2008/11/22 Walter Tadeu Nogueira da Silveira <[EMAIL PROTECTED]>

> Concordo com o João
>
> Aliás, postei enganado o IPP. Queria por o IIP que a conta também dá 50. O
> PPP dá 10. Pareceu a todos que a ordem não faria diferença.
> A parte boa foi que apesar do gabarito oficial, nenhum aluno concordou.
> Obrigado a todos!
>
> 2008/11/22 João Luís <[EMAIL PROTECTED]>
>
>>  Com dois pares e um ímpar, a soma dos três não será par.
>>
>> Para mim, a solução desse problema é a seguinte:
>>
>> Para que a soma dos três seja para, podemos escolher "nenhum ímpar e três
>> pares" (10 modos) ou "dois ímpares e um par" (50 modos), não importando a
>> ordem da escolha, em virtude da comutatividade da adição.
>>
>> Portanto, teremos 60 escolhas.
>>
>> Um abraço a todos,
>>
>> João Luís.
>>
>>  - Original Message -
>> *From:* Antonio Neto <[EMAIL PROTECTED]>
>> *To:* obm-l@mat.puc-rio.br
>>   *Sent:* Saturday, November 22, 2008 10:25 AM
>> *Subject:* RE: [obm-l] Contagem
>>
>> Oi,
>> receio que haja alguns pequenos enganos. No caso PPP, tudo bem, mas o
>> outro caso nao eh PPI, mas PII, o que nao acarretaria problemas de contas se
>> tivesse sido resolvido corretamente. Ele se divide em tres casos, PII, PIP e
>> IPP, logo o seu 50 eh na verdade 50*3 = 150. Acho que agora estah tudo
>> certinho. Amplexos, olavo
>>
>>
>> Antonio *Olavo* da Silva Neto
>>
>>
>>
>>
>> --
>> Date: Fri, 21 Nov 2008 20:22:26 -0200
>> From: [EMAIL PROTECTED]
>> To: obm-l@mat.puc-rio.br
>> Subject: [obm-l] Contagem
>>
>> O problema abaixo foi trazido por um aluno. Eis a solução encontrada pela
>> turma:
>>
>> "O número de possibilidades de escolha de 3 números naturais distintos de
>> 1 a 10, de modo que sua soma seja sempre par, é:"
>>
>>1. 120
>>2. 220
>>3. 150
>>4. 290
>>5. 160
>>
>> SOLUÇÃO. Supõe-se que são cartões com os números onde:
>> Pares: 2, 4, 6, 8 e 10
>> Ímpares: 1, 3, 5, 7, 9
>> Para que a escolha dos três números dê soma par, deve-se ter: P P P ou I P
>> P
>> a) P P P temos: C(5,3) = 10
>> b) I P P temos: C(5,1) x C(5,2) = 5 x 10 = 50
>> Total de 10 + 50 = 60 possibilidades.
>> Ficaram felizes, mas a resposta apontava 160. Não consegui mostrar o erro
>> a eles. Alguém poderia dar uma ajuda? Grato.
>>
>>
>> Walter Tadeu Nogueira da Silveira
>>
>> --
>> Get news, entertainment and everything you care about at Live.com. Check
>> it out! 
>>
>>
>
>
> --
> Walter Tadeu Nogueira da Silveira
> www.professorwaltertadeu.mat.br
>


Re: [obm-l] Jogo dos 4 bits

2008-11-18 Por tôpico Fellipe Rossi
Com no máximo cinco:
1a - Usuário: 

-  Se der 0 ou 4, a resposta é imediata. FIM na 1a (se der 4) ou 2a (se der
0)

-  Se der 1 ou 3, basta variar o bit nas 4 posições (no máximo) totalizando
4 tentativas na pior das hipóteses. FIM no máximo na 5a etapa

-  Se der 2:
2a - Usuário insere dois 0 e dois 1.

- Se der 0 ou 4, a resposta é imediata. FIM na 2a (se der 4) ou 3a. (se der
0)

- Senão, a resposta só pode ser 2. Então:
3a. Usuário escolhe um par 01 e inverte.

- Se der 0 ou 4, a resposta é imediata. FIM na 3a. (se der 4) ou 4a. (se der
0)

- Se der 2 novamente, significa que tanto no par que foi mexido quanto no
par que não foi mexido havia 1 bit na posição correta:

4a. Inverte o 0 do primeiro par com o 1 do segundo par
- Vai dar 0 ou 4, a resposta é imediata. FIM na 4a.(se der 4) ou 5a (se der
0)

Agora resta saber se dá com 4 hehe

2008/11/18 Felipe <[EMAIL PROTECTED]>

> é entao, fiquei na duvida se podia usar entropia aí, pq eh um pouco
> diferente, mas com 6 eh tranquilo acertar
>  problema eh provar mesmo quantos sao ehhe
>
> 2008/11/18 Maurício Collares <[EMAIL PROTECTED]>
>
> Não existe maneira de fazer só com 4 tentativas. Suponha que o
>> computador pode prever o que você vai falar (isso não é nada irreal,
>> pois se o computador escolhe os bits aleatoriamente e uniformemente, a
>> chance de ele escolher o correspondente a "adivinhar" é não nula).
>> Você tenta "0001" e o computador, só de maldade, responde "1". Você
>> não sabe qual dos 4 dígitos está errado, mas o fato é que você só tem
>> mais três tentativas pra acertar segundo sua conjectura:
>>
>> * Se você tentar mudar apenas um dígito por tentativa (em relação a
>> combinação original), o computador adivinha qual dígito você não
>> tentou e escolhe ele como o errado, de modo que você só vai poder
>> acertar de quinta.
>> * Se você mudar mais de um dígito em uma das três tentativas que
>> sobrou e, por consequencia disso, conseguir mudar todos os dígitos (se
>> não mudar todos, faz a mesma coisa que no caso anterior), você vai ter
>> dois dígitos que sempre são mudados juntos em relação a configuração
>> original. Então o computador adivinha qual par sempre é testado
>> conjuntamente e escolhe um elemento do par como errado. Então, em
>> todas as tentativas, ou você não muda o dígito errado, ou muda o
>> dígito errado pra um dígito certo e muda um dígito certo pra um dígito
>> errado.
>>
>> Segue-se que, no pior caso, 4 lances não bastam. É fácil fazer com 6
>> lances (basta tentar uma configuração inicial qualquer e mudar os bits
>> um por um para conseguir informação sobre eles: em cinco lances, vc
>> tem toda a informação necessária, e aí dá pra acertar no sexto), mas
>> não pensei como fazer pra provar que dá (ou que não dá) pra fazer com
>> cinco.
>>
>> --
>> Abraços,
>> Maurício
>>
>> 2008/11/18 Felipe <[EMAIL PROTECTED]>:
>> > por entropia deveria ser -log_2 ^(1/(2^4))=log_2^(2^4)=4 bits
>> > certo?
>> >
>> > 1a-q tal poe sem perda de generalidade 
>> > descobre que sao 2 zeros e 2 uns (2 acertos)
>> > 2a-depois poe 0111 (3 acertos)
>> > 3a- 0001
>> > 4a-
>> >
>> > ah fiz rapido , alguem deve achar uma maneira de no máximo 4.
>> >
>> >
>> >
>> > 2008/11/18 Douglas Ribeiro Silva <[EMAIL PROTECTED]>
>> >>
>> >> O jogo dos 4 bits consiste no computador escolher um número de 4 bits
>> >> e o usuário tentar adivinhar. Para cada palpite do usuário o
>> >> computador retorna quantos bits ele acertou.
>> >>
>> >> Ex: o computador escolhe 0101
>> >>
>> >> Usuario: 
>> >> PC:2
>> >> Usuario: 0100
>> >> PC: 3
>> >> Usuario: 
>> >> PC: 2
>> >> Usuario: 0111
>> >> PC: 1
>> >> Usuario: 0101
>> >> PC: 4
>> >>
>> >> Qual a melhor estratégia para o jogo? O jogador deve sempre trocar a
>> >> quantidade de dígitos que o computador indicar? Qual a quantidade
>> >> máxima que um usuário inteligente gastaria para acertar o numero?
>> >>
>> >>
>> =
>> >> Instruções para entrar na lista, sair da lista e usar a lista em
>> >> http://www.mat.puc-rio.br/~obmlistas/obm-l.html
>> >>
>> =
>> >
>> >
>>
>> =
>> Instruções para entrar na lista, sair da lista e usar a lista em
>> http://www.mat.puc-rio.br/~obmlistas/obm-l.html
>> =
>>
>
>


Re: [obm-l] Jogo dos 4 bits

2008-11-18 Por tôpico Fellipe Rossi
Ainda pergunto: o computador retorna quantos bits estão certos ou quantos
bits estão certos nas posicoes certas?
Por exemplo. Se o computador escolhe 0001

Eu chuto 1000

Ele me retorna 4 (3 zeros e 1 um) ou 2 (apenas 2 zeros na posicao correta)?

Isso faz toda a diferença e não ficou claro no enunciado.

2008/11/18 Maurício Collares <[EMAIL PROTECTED]>

> Não existe maneira de fazer só com 4 tentativas. Suponha que o
> computador pode prever o que você vai falar (isso não é nada irreal,
> pois se o computador escolhe os bits aleatoriamente e uniformemente, a
> chance de ele escolher o correspondente a "adivinhar" é não nula).
> Você tenta "0001" e o computador, só de maldade, responde "1". Você
> não sabe qual dos 4 dígitos está errado, mas o fato é que você só tem
> mais três tentativas pra acertar segundo sua conjectura:
>
> * Se você tentar mudar apenas um dígito por tentativa (em relação a
> combinação original), o computador adivinha qual dígito você não
> tentou e escolhe ele como o errado, de modo que você só vai poder
> acertar de quinta.
> * Se você mudar mais de um dígito em uma das três tentativas que
> sobrou e, por consequencia disso, conseguir mudar todos os dígitos (se
> não mudar todos, faz a mesma coisa que no caso anterior), você vai ter
> dois dígitos que sempre são mudados juntos em relação a configuração
> original. Então o computador adivinha qual par sempre é testado
> conjuntamente e escolhe um elemento do par como errado. Então, em
> todas as tentativas, ou você não muda o dígito errado, ou muda o
> dígito errado pra um dígito certo e muda um dígito certo pra um dígito
> errado.
>
> Segue-se que, no pior caso, 4 lances não bastam. É fácil fazer com 6
> lances (basta tentar uma configuração inicial qualquer e mudar os bits
> um por um para conseguir informação sobre eles: em cinco lances, vc
> tem toda a informação necessária, e aí dá pra acertar no sexto), mas
> não pensei como fazer pra provar que dá (ou que não dá) pra fazer com
> cinco.
>
> --
> Abraços,
> Maurício
>
> 2008/11/18 Felipe <[EMAIL PROTECTED]>:
> > por entropia deveria ser -log_2 ^(1/(2^4))=log_2^(2^4)=4 bits
> > certo?
> >
> > 1a-q tal poe sem perda de generalidade 
> > descobre que sao 2 zeros e 2 uns (2 acertos)
> > 2a-depois poe 0111 (3 acertos)
> > 3a- 0001
> > 4a-
> >
> > ah fiz rapido , alguem deve achar uma maneira de no máximo 4.
> >
> >
> >
> > 2008/11/18 Douglas Ribeiro Silva <[EMAIL PROTECTED]>
> >>
> >> O jogo dos 4 bits consiste no computador escolher um número de 4 bits
> >> e o usuário tentar adivinhar. Para cada palpite do usuário o
> >> computador retorna quantos bits ele acertou.
> >>
> >> Ex: o computador escolhe 0101
> >>
> >> Usuario: 
> >> PC:2
> >> Usuario: 0100
> >> PC: 3
> >> Usuario: 
> >> PC: 2
> >> Usuario: 0111
> >> PC: 1
> >> Usuario: 0101
> >> PC: 4
> >>
> >> Qual a melhor estratégia para o jogo? O jogador deve sempre trocar a
> >> quantidade de dígitos que o computador indicar? Qual a quantidade
> >> máxima que um usuário inteligente gastaria para acertar o numero?
> >>
> >>
> =
> >> Instruções para entrar na lista, sair da lista e usar a lista em
> >> http://www.mat.puc-rio.br/~obmlistas/obm-l.html
> >>
> =
> >
> >
>
> =
> Instruções para entrar na lista, sair da lista e usar a lista em
> http://www.mat.puc-rio.br/~obmlistas/obm-l.html
> =
>


Re: [obm-l] Jogo dos 4 bits

2008-11-18 Por tôpico Fellipe Rossi
nao ficou muito claro. O PC retorna so os bits que estão nas posições
corretas né?

2008/11/18 Douglas Ribeiro Silva <[EMAIL PROTECTED]>

> O jogo dos 4 bits consiste no computador escolher um número de 4 bits
> e o usuário tentar adivinhar. Para cada palpite do usuário o
> computador retorna quantos bits ele acertou.
>
> Ex: o computador escolhe 0101
>
> Usuario: 
> PC:2
> Usuario: 0100
> PC: 3
> Usuario: 
> PC: 2
> Usuario: 0111
> PC: 1
> Usuario: 0101
> PC: 4
>
> Qual a melhor estratégia para o jogo? O jogador deve sempre trocar a
> quantidade de dígitos que o computador indicar? Qual a quantidade
> máxima que um usuário inteligente gastaria para acertar o numero?
>
> =
> Instruções para entrar na lista, sair da lista e usar a lista em
> http://www.mat.puc-rio.br/~obmlistas/obm-l.html
> =
>


Re: [obm-l] Re: [obm-l] picaretagem no ensino da matemática

2007-07-04 Por tôpico Fellipe Rossi

Lamentável

Em 04/07/07, Rogerio Ponce <[EMAIL PROTECTED]> escreveu:


Ola' pessoal,
ainda bem que ele nao refinou os macetes:

Por exemplo, quanto vale 199 / 995?
Ora, basta "cortar" os "99" de cima e de baixo, de modo que o resultado e'
1/5 ( ( pode conferir ! ).

Da mesma forma, calcular 424/742 tambem e' muito facil: basta simplificar
o "42" de cima com o "42" de baixo, e o resultado e' 4/7.

Simples, nao?

[]'s
Rogerio Ponce

*João Luís Gomes Guimarães <[EMAIL PROTECTED]>* escreveu:


Ou então a raiz de 784, por exemplo... é só "desprezar" o 8, e fazer a
raiz
de 7 (?) e a raiz de 4...

é muito pilantra que há nesse mundo, mesmo...

- Original Message -
From: "Romildo Franco"
To:
Sent: Wednesday, July 04, 2007 12:58 PM
Subject: Re: [obm-l] picaretagem no ensino da matemática


> Manda ele achar a raiz de 962 agora.
>
=
> Instruções para entrar na lista, sair da lista e usar a lista em
> http://www.mat.puc-rio.br/~nicolau/olimp/obm-l.html
>
=


=
Instruções para entrar na lista, sair da lista e usar a lista em
http://www.mat.puc-rio.br/~nicolau/olimp/obm-l.html
=


 ------
Novo Yahoo! Cadê? <http://yahoo.com.br/oqueeuganhocomisso+> - Experimente
uma nova busca.





--
Fellipe Rossi


Re: [obm-l] Desafio - Análise Real

2007-07-03 Por tôpico Fellipe Rossi

Caros colegas,

Será que a resolução abaixo estaria correta?

Talvez, usando a informação das somas dos módulos de b_n do enunciado, fique
mais simples assim:

___

Como (a_n) converge para 0, dado e > 0, |a_n| < e/k para todo n natural
positivo.



Da Desigualdade Triangular, temos:



|c_n|  <=  |a_1|.|b_n| + |a_2|.|b_(n-1)| + ... + |a_n|.|b_1|


Como , |an| < e/k, para todo n natural positivo, temos:


|c_n|  <  (e/k).|b_n| + (e/k).|b_(n-1)| + ... + (e/k).|b_1|. = (e/k).(|b_n|
+ |b_(n-1)| + ... + |b_1|)  para todo n natural positivo.



Mas, do enunciado, temos |b_n| + |b_(n-1)| + ... + |b_1| < k para todo n
natural positivo.



Portanto, |cn| < (e/k).k = e, para todo natural positivo e, portanto, (c_n)
converge para 0.


__


E então? Está correto?

Grande abraço,

Fellipe Rossi


Em 29/06/07, Nicolau C. Saldanha <[EMAIL PROTECTED]> escreveu:


On Thu, Jun 28, 2007 at 01:49:20PM -0300, Fellipe Rossi wrote:
> Sejam (a_n) e (b_n) duas seqüências de números reais convergentes para
zero e
> suponha que existe k > 0 tal que |b_1| + |b_2| + |b_3| + ... + |b_n| < k
para
> todo n pertencente a IN*. Mostre que a seqüência  (c_n) definida por c_n
=
> a_1.b_n + a_2.b_n-1 + ... + a_n.b_1 converge para zero.
>
> Notação: a_k = termo de índice k da seqüência a.

As seqüências a_k e b_k são limitadas:
suponha que |a_k|, |b_k| < B para todo k.

Dado e > 0 seja N1 tal que n > N1 -> |b_(N1+1)|+...+|b_n| < e/(2B).
Seja C = |b_1|+|b_2|+...+|b_N1|.
Seja N2 tal que n > N2 -> |a_n| < e/(2C).

Tome N = N1+N2 e n > N.
|c_n| <= |a_1||b_n| + ... + |a_(n-N1)| |b_(N1+1)| +
 |a_(n+1-N1)||b_N1| + ... + |a_n| |b_1|

Na primeira linha temos |a_k| < B.
Temos n+1-N1 > N2 donde na segunda linha temos |a_k| < e/(2C).
Assim

|c_n| <= B(|b_n| + ... + |b_(N1+1)|) +
 (e/(2C))(|b_N1| + ... + |b_1|)
  < Be/(2B) + Ce/(2C) = e

concluindo a demonstração.

[]s, N.

=
Instruções para entrar na lista, sair da lista e usar a lista em
http://www.mat.puc-rio.br/~nicolau/olimp/obm-l.html
=


Re: [obm-l] RES: [obm-l] RE: [obm-l] Probabilidade do triângulo

2007-06-30 Por tôpico Fellipe Rossi

Pelo que eu entendi do enunciado, os segmentos "determinados" seriam AC, CD
e DB. Não? Na solução você considera AC = x, AD = y e DB = 1 -y, certo? Não
seria talvez AC = x, CD = y e DB = 1 - y - x?

Pelo que eu entendi na sua resolução, y nao tem que ser menor que 1/2.
Poderíamos ter, p.ex., y = 2/3, x = 1/3 por exemplo e mesmo assim AC, CD e
DB seriam iguais a 1/3 e formariam um triangulo equilátero. Certo?

De qqer forma achei muito legal a solução.


Em 30/06/07, carry_bit <[EMAIL PROTECTED]> escreveu:


Obrigado, pela resolução!

-Mensagem original-
De: [EMAIL PROTECTED] [mailto:[EMAIL PROTECTED] Em nome
de Ralph Teixeira
Enviada em: segunda-feira, 18 de junho de 2007 23:26
Para: obm-l@mat.puc-rio.br
Assunto: [obm-l] RE: [obm-l] Probabilidade do triângulo



   -Original Message-
   From: Ralph Teixeira
   Sent: Thu 6/7/2007 3:57 PM
   To: obm-l@mat.puc-rio.br
   Cc:
   Subject: RE: [obm-l] Probabilidade do triângulo


   Sem perda de generalidade, suponha que o comprimento de AB eh 1.
Sejam AC=x e AD=y, tambem sem perda de generalidade.

   Agora, este negocio de "marcados ao acaso" eh mais ambiguo do que
parece -- existem varias maneiras diferentes de escolher os pontos ao
acaso,
que podem dar resultados diferentes. A maneira mais comum de interpretar
isso (pontos independentes, distribuicao uniforme) dah o seguinte
argumento:

   Considere o ponto (x,y) no plano cartesiano. Como 0<=x<=1 e
0<=y<=1,
este ponto estah no quadrado de lado 1 com vertice na origem (faca a
figura!). Quais destas escolhas sao "validas"? Bom, uma escolha eh valida
se
os 3 segmentos sao menores que 1/2 (pois entao o maior serah menor que a
soma dos outros dois).

   Se x<=y, os segmentos sao x, y-x e 1-y. Assim, queremos x<=1/2,
y-x<=1/2 e y>=1/2. Marque estas regioes no quadrado dentro de y>=x.
   Se x>=y, a situacao eh simetrica: queremos agora x>=1/2, x-y>=1/2 e
y<=1/2. A regiao "valida" eh entao algo assim (viva arte ASCII!!):

   0=x   1/2=x1=x
   ooox y=1
   ooxx
   oxxx
   
   ooox
   ooxx
   oxxx
    y=1/2
   oooo
   oooxxxoo
   oooxxooo
   ooox
   oooo
   oooxxxoo
   oooxxooo
   ooox  y=0

   A interpretacao usual de "escolher ao acaso" eh de que a
probabilidade de o ponto escolhido estar numa area seria proporcional a
esta
area (distribuicao uniforme). Entao a probabilidade pedida eh a area da
regiao com x sobre a area total do quadrado. Dah 1/4=25%.

   Abraco,
 Ralph

   -Original Message-
   From: [EMAIL PROTECTED] on behalf of carry_bit
   Sent: Sat 5/19/2007 8:57 PM
   To: obm-l@mat.puc-rio.br
   Cc:
   Subject: [obm-l] Probabilidade do triângulo



   Olá integrantes da obm-l,



   Eu me deparei com o seguinte problema e não consegui
resolver!



   * Dado um segmento de reta AB qualquer, dois pontos
(C e D) são marcados ao acaso nesse segmento. Qual é a probabilidade de os
três segmentos assim formados poderem constituir um triângulo?





   Agradeço, Carry_bit


=
Instruções para entrar na lista, sair da lista e usar a lista em
http://www.mat.puc-rio.br/~nicolau/olimp/obm-l.html
=


=
Instruções para entrar na lista, sair da lista e usar a lista em
http://www.mat.puc-rio.br/~nicolau/olimp/obm-l.html
=





--
Fellipe Rossi


Re: [obm-l] confirmar probabilidade

2007-06-30 Por tôpico Fellipe Rossi

Sim

Em 30/06/07, Tio Cabri st <[EMAIL PROTECTED]> escreveu:


Bom dia, o exercicio eh mais ou menos assim,

7 moedas de valor x
8 moedas de valor y
e
5 moedas de valor z
Qual a probabilidade de ao tirarmos 3 moedas existir uma e só uma de valor
x?
Minha resposta foi  91/190 está certo?
Abraços
Tio Cabri

=
Instruções para entrar na lista, sair da lista e usar a lista em
http://www.mat.puc-rio.br/~nicolau/olimp/obm-l.html
=





--
Fellipe Rossi


[obm-l] Desafio - Análise Real

2007-06-28 Por tôpico Fellipe Rossi
Caros colegas,
Estou tendo dificuldades para resolver uma questão de Análise - mais 
precisamente, seqüências.
Pesquisei em alguns livros e até sites mas não encontrei nenhuma dica que 
pudesse me ajudar. O problema é o seguinte:

Sejam (a_n) e (b_n) duas seqüências de números reais convergentes para zero e 
suponha que existe k > 0 tal que |b_1| + |b_2| + |b_3| + ... + |b_n| < k para 
todo n pertencente a IN*. Mostre que a seqüência  (c_n) definida por c_n = 
a_1.b_n + a_2.b_n-1 + ... + a_n.b_1 converge para zero. 


Notação: a_k = termo de índice k da seqüência a.

[obm-l] Re: [obm-l] Combinatória

2004-09-23 Por tôpico Fellipe Rossi



52 cartas, onde 12 sao figuras
20 pares 
20 impares
 
PAR FIG FIG x P(3,2) = 20/52 * 12/51 * 11/50 * 
3
 
Fui
 
 

  - Original Message - 
  From: 
  Daniela Yoshikawa 
  To: [EMAIL PROTECTED] 
  Sent: Thursday, September 23, 2004 8:32 
  PM
  Subject: [obm-l] Combinatória
  
  Ai vai uma de Combinatória.
   
  De um baralho com 52 cartas, sorteia-se 3 cartas. Quantas são as chances 
  de se ter um número par e duas figuras?
  
  
  Yahoo! 
  Messenger 6.0 - jogos, emoticons sonoros e muita diversão. Instale 
agora!


Re: Alerta de spam:Re: [obm-l] AJUDA!!!!!!!

2004-06-14 Por tôpico Fellipe Rossi



e o ZERO?

  - Original Message - 
  From: 
  [EMAIL PROTECTED] 
  
  To: [EMAIL PROTECTED] 
  Sent: Sunday, June 13, 2004 7:30 PM
  Subject: Alerta de spam:Re: [obm-l] 
  AJUDA!!!
  Em uma 
  mensagem de 13/6/2004 18:37:53 Hora padrão leste da Am. Sul, [EMAIL PROTECTED] escreveu: 
  )quantos números de 3 algarismos podemos fomar usando pelo 
menos 2 algarismo repetido a)38 b)252 c)300 d)414 
e)nda Podemos ter 3 casos de numeros: 1º 
  caso: _ r r 2º caso: r r _ 3º caso: r _ r 1º caso: Os 
  numeros sao: _ 11 (9 possiveis numeros, pois devemos desconsiderar o 
  zero) _ 22 (9 possiveis numeros, pois devemos desconsiderar o 
  zero) _ 33 
  (9 possiveis numeros, 
  pois devemos desconsiderar o zero) _ 44 (9 possiveis numeros, pois devemos 
  desconsiderar o zero) 
  _ 55 (9 
  possiveis numeros, pois devemos desconsiderar o zero) _ 66 (9 possiveis numeros, pois devemos 
  desconsiderar o zero) 
  _ 77 (9 
  possiveis numeros, pois devemos desconsiderar o zero) _ 88 (9 possiveis numeros, pois devemos 
  desconsiderar o zero) 
  _ 99 (9 
  possiveis numeros, pois devemos desconsiderar o zero) Sub-total = 9*9 
  = 81 numeros (lembrando que os numeros 111, 222, 333, ..., 999 ja foram 
  incluidos acima) 
  2º caso: 
  Os numeros sao: 1 1 _  (9 possiveis numeros, pois devemos 
  desconsiderar 1) 2 2 _  (9 possiveis numeros, pois devemos 
  desconsiderar 2) 3 3 _  (9 possiveis numeros, pois devemos 
  desconsiderar 3) 4 4 _  (9 possiveis numeros, pois devemos 
  desconsiderar 4) 5 5 _  (9 possiveis numeros, pois devemos 
  desconsiderar 5) 6 6 _  (9 possiveis numeros, pois devemos 
  desconsiderar 6) 7 7 _  (9 possiveis numeros, pois devemos 
  desconsiderar 7) 8 8 _  (9 possiveis numeros, pois devemos 
  desconsiderar 8) 9 9 _  (9 possiveis numeros, pois devemos 
  desconsiderar 9) Sub-total = 9*9 = 81 numeros 1º caso: Os numeros sao: 
  1 _ 1 (9 possiveis numeros, pois devemos desconsiderar 1) 2 _ 2 (9 
  possiveis numeros, pois devemos desconsiderar 2) 3 _ 3 (9 possiveis 
  numeros, pois devemos desconsiderar 3) 4 _ 4 (9 possiveis numeros, pois 
  devemos desconsiderar 4) 5 _ 5 (9 possiveis numeros, pois devemos 
  desconsiderar 5) 6 _ 6 (9 possiveis numeros, pois devemos desconsiderar 6) 
  7 _ 7 (9 possiveis numeros, pois devemos desconsiderar 7) 8 _ 8 (9 
  possiveis numeros, pois devemos desconsiderar 8) 9 _ 9 (9 possiveis 
  numeros, pois devemos desconsiderar 9) Sub-total = 9*9 = 81 numeros 
  Logo o total = 3*81 = 243 (n.d.a) 
  Numa reunião de jovens, há 10 rapazes e 5 raparigas. então o 
número de grupos de 5 jovens que podem ser formados, tendo cada grupo no 
máximo 1 rapaz´? a)50 b)51 c)52 d)60 e)nda[ 
  Podemos ter 2 casos: 1º caso: 0 rapaz e 5 mocas 
  > 1 grupo 2º caso: 1 rapaz e 4 mocas > Para escolher um 
  rapaz temos C(10,1) possibilidades e para escolher 4 mocas temos C(5,4) 
  possibilidades, logo ha C(10,1)*C(5,4) = 50 possibilidades de se escolher 
  1 rapaz e 4 mocas. 
  Total: 1º caso + 2º caso = 1 + 50 = 51 grupos 


Re: Alerta de spam:[obm-l] AJUDA!!!!!!!

2004-06-13 Por tôpico Fellipe Rossi



a 1) ja enviei
 
a 2)
 
Com nenhum homem:1 
grupo
com 1 homem: 10xC(5,4) = 50
 
Total = 51 grupos.

  - Original Message - 
  From: 
  TSD 
  To: [EMAIL PROTECTED] 
  Sent: Sunday, June 13, 2004 6:34 PM
  Subject: Alerta de spam:[obm-l] 
  AJUDA!!!
  
  )quantos números de 3 algarismos podemos fomar 
  usando pelo menos 2algarismo 
  repetidoa)38b)252c)300d)414e)ndaNuma reunião 
  de jovens, há 10 rapazes e 5 raparigas. então o númerode grupos de 5 
  jovens que podem ser formados, tendo cada grupo nomáximo 1 
  rapaz´?a)50b)51c)52d)60e)nda[


[obm-l] Re: [obm-l] dúvidazinha

2004-06-13 Por tôpico Fellipe Rossi



AAB - 9x1x9 = 81
ABA - 9x9x1 = 81
BAA - 9x9x1 = 81
 
AAA = 9x1x1 = 9
 
Somando tudo dá 252;
 
ou a letra b ta escrita errada ou eh NDA, ou eu 
errei ;P
 
Rossi

  - Original Message - 
  From: 
  TSD 
  To: [EMAIL PROTECTED] 
  Sent: Sunday, June 13, 2004 6:02 PM
  Subject: [obm-l] dúvidazinha
  
  quantos números de 3 algarismos podemos fomar 
  usando pelo menos 2 algarismo repetido.
   
  a)38 b)25 c)300 d)414 e)nda
   
   


[obm-l] Re:

2004-06-12 Por tôpico Fellipe Rossi
três numeros consecutivos multiplos de 7 = (x-7, x, x+7)
a soma = x-7+x+x+7 = 273 => 3x = 273, então x= 91.
Logo os numeros são 84, 91, 98.

Letra A

Rossi
- Original Message -
From: "elton francisco ferreira" <[EMAIL PROTECTED]>
To: <[EMAIL PROTECTED]>
Sent: Saturday, June 12, 2004 8:22 PM


> A soma de três múltiplos consecutivos de 7 é 273. O
> maior desses números é um número:
>
> por
> impar
> multiplo de 3
> multiplo de 4
>
>
> __
>
> Participe da pesquisa global sobre o Yahoo! Mail:
> http://br.surveys.yahoo.com/global_mail_survey_br
> =
> Instruções para entrar na lista, sair da lista e usar a lista em
> http://www.mat.puc-rio.br/~nicolau/olimp/obm-l.html
> =
>
>


=
Instruções para entrar na lista, sair da lista e usar a lista em
http://www.mat.puc-rio.br/~nicolau/olimp/obm-l.html
=


Re: [obm-l] Probleminha

2004-06-10 Por tôpico Fellipe Rossi



20.000 x 4 = total (4 rodas)
porem pra esse percurso 5 foram 
utilizadas
logo sao 8/5 = 16000 por roda

  - Original Message - 
  From: 
  Fábio Bernardo 
  
  To: OBM 
  Sent: Thursday, June 10, 2004 12:50 
  PM
  Subject: [obm-l] Probleminha
  
  Pessoal, tô enrolado nesse. Ajudem-me por 
  favor.
   
  Em um carro foram usados os 4 pneus mais o 
  estepe, rodando igualmente a mesma quilometragem. Após o carro ter percorrido 
  2km, cada pneu foi usado por:
  a) 2km
  b) 16000km
  c) 12000km
  d) 8000km
  e) 4000km
   
   


[obm-l] Re: [obm-l] Continuidade - Exercício

2004-06-08 Por tôpico Fellipe Rossi
Title: Re: [obm-l] Continuidade - Exercício



Muito obrigado! Eu tenho prova disso amanha! vc 
ajudou bastante!! :)
 
Eu posso dizer que lim(x->0) (e^x - 1)/x = 1 é 
um limite fundamental?
ou numa prova eu precisaria provar 
isso?
 
Abraços
Rossi

  - Original Message - 
  From: 
  Claudio Buffara 
  To: [EMAIL PROTECTED] 
  Sent: Tuesday, June 08, 2004 4:14 
PM
  Subject: Re: [obm-l] Continuidade - 
  Exercício
  on 08.06.04 14:44, Fellipe Rossi at [EMAIL PROTECTED] 
  wrote:
  Caros amigos da lista, espero que 
possam me ajudar ;)QUESTÃO:Determine a e b para que f(x) seja contínua em 
R.onde 
f(x)=(e^ax - 
1)(x^4 +2) , para x<0x^5 + 6x^3 + 
9xa*sen(x*pi) + b 
 para 0<=x<=1/2 8x^3 - 4x^2 - 2x + 1 
.  para x>1/24x^4 - 
4x^3 + 5x^2 - 4x + 1Eu fiz uma das relações entre a e b, vendo os limites laterais da 
terceira equação, que, simplificando, ficou: (2x+1) / (x^2 
+1)Assim, a+b = 
8/5Porém quando fui 
aplicar a definição de continuidade na primeira equação cai em um limite 
indeterminado para os valores de 0 pela esquerda e não consegui levantar a 
indeterminação (lembrando que não posso usar L'hôpital pois o professor 
vetou).Será que alguem 
ai conseguiria tirar a indeterminação? ou mesmo resolver de outra 
maneira?Ahh! a resposta 
é a=72/55 e b=16/55 (o que torna válida a relação a+b= 8/5 
).Agradeço desde 
já!Abraços,RossiO que voce quer eh provar 
que lim(x->0) (e^(ax) - 1)/x = 1 sem usar L'Hopital.Soh que isso 
depende de como voce define a funcao exponencial.Por exemplo, uma 
forma eh definir a funcao log:(0,+infinito) -> R como sendo:log(x) = 
Integral(1..x) dt/te depois provar que, para quaisquer x, y positivos, 
vale log(xy) = log(x) + log(y).Dai decorre que log eh uma bijecao 
infinitamente diferenciavel tal que:.log(1) = 0;existe um unico 
numero real, representado por e, tal que log(e) = 1; log'(x) = 1/x para 
todo x > 0.Em seguida, definimos a funcao exp:R -> 
(0,+infinito) como sendo a inversa da log, de forma que exp(0) = 1, exp(1) = 
e, em geral, exp(x) = e^x.Finalmente, a derivada de exp em x = 0 eh 
igual a:exp'(0) = lim(h->0) (exp(0+h) - exp(0))/h = lim(h->0) (e^h 
- 1)/h.Mas, como, para todo x real vale log(exp(x)) = x, a regra da 
cadeia implica que, para todo x: log'(exp(x))*exp'(x) = 1 ==> 
(1/exp(x))*exp'(x) = 1 ==> exp'(x) = exp(x).Em particular, 
exp'(0) = exp(0) = 1, ou seja, lim(h->0) (e^h - 1)/h = 1Eh facil 
ver, a partir disso, que se a <> 0, entao lim(x->0) (e^(ax) - 1)/x 
= a, bastando apenas fazer a mudanca de variavel y = 
ax.[]s,Claudio.


[obm-l] Continuidade - Exercício

2004-06-08 Por tôpico Fellipe Rossi



Caros amigos da lista, espero que possam me ajudar 
;)
 
QUESTÃO:
Determine a e b para que f(x) seja contínua em 
R.
 
onde f(x)=
 
(e^ax - 1)(x^4 +2) , para 
x<0
x^5 + 6x^3 + 9x
 
a*sen(x*pi) + b  para 
0<=x<=1/2
 
  8x^3 - 
4x^2 - 2x + 1 .  para 
x>1/2
4x^4 - 4x^3 + 5x^2 - 4x + 
1
 
Eu fiz uma das relações entre a e b, vendo os 
limites laterais da terceira equação, que, simplificando, ficou: (2x+1) / (x^2 +1)
 
Assim, a+b = 8/5
 
Porém quando fui aplicar a definição de 
continuidade na primeira equação cai em um limite indeterminado para os valores 
de 0 pela esquerda e não consegui levantar a indeterminação (lembrando que não 
posso usar L'hôpital pois o professor vetou).
 
Será que alguem ai conseguiria tirar a 
indeterminação? ou mesmo resolver de outra maneira?
 
Ahh! a resposta é a=72/55 e b=16/55 (o que torna 
válida a relação a+b= 8/5 ).
 
Agradeço desde já!
Abraços,
Rossi
 
 
 
 


Re: [obm-l] aritmetica

2004-06-07 Por tôpico Fellipe Rossi
V(t) = 38t + 47t - 21t,  V em litros e t em minutos.
V(t) = 64t
V = V(680) = 64*680 = 43520.

RESPOSTA: V = 43520 litros

- Original Message -
From: "elton francisco ferreira" <[EMAIL PROTECTED]>
To: <[EMAIL PROTECTED]>
Sent: Sunday, June 06, 2004 7:49 PM
Subject: [obm-l] aritmetica


> Um reservatório é alimentado por duas torneiras: a
> primeira dá 38 litros por minuto e a segunda, 47. A
> saída de água é por um orifício que deixa passar 21
> litros por minuto, deixando abertas as torneiras e o
> orifício, o reservatório se enche em 680 minutos. Qual
> é a sua capacidade?
>
> __
>
> Participe da pesquisa global sobre o Yahoo! Mail:
> http://br.surveys.yahoo.com/global_mail_survey_br
> =
> Instruções para entrar na lista, sair da lista e usar a lista em
> http://www.mat.puc-rio.br/~nicolau/olimp/obm-l.html
> =
>


=
Instruções para entrar na lista, sair da lista e usar a lista em
http://www.mat.puc-rio.br/~nicolau/olimp/obm-l.html
=


Re: [obm-l] OBM 2004 - Nivel 3

2004-06-05 Por tôpico Fellipe Rossi
Faça 2 leis dos cossenos
1 no triângulo BEC e outra no triãngulo AEB
Como o angulo AÊB = 180-BÊC, os seus cossenos são simétricos.

I) (15/2)^2 = 100 + 25 - 100cosY
   225/4 = 125 - 100cosY
cosY = 11/16

II) x^2 = 4+25 - 20cos(180-Y)
x^2 = 29+20cosY => x^2=29 + 20(11/16)

Achei x= sqrt(171)/2

Não conferi as contas ;)

Abraços!

- Original Message -
From: "Vitor Dias" <[EMAIL PROTECTED]>
To: <[EMAIL PROTECTED]>
Sent: Sunday, June 06, 2004 1:22 AM
Subject: [obm-l] OBM 2004 - Nivel 3


> Fiz a prova...achei bem legal, e muito bem elaborada! foi um otimo
treinamento,
> jah q vou prestar concurso para o CN e para EPCAr...(alem do que sou um
> eterno apaixonado pela matematica hehe)
> bem, vou deixar uma questao legal de geometria plana, tirada do livro
Challenging
> Problems in Geometry, de Alfred S. Posamentier e Charles T. Salkind:
>
> As diagonais AC e BD de um quadrilatero ADCB se encontram em E. Se AE=2,
> BE=5, CE=10, DE=4 e BC=15/2, calcule AB.
>
> Divirtam-se!!!
>
>
> =
> Instruções para entrar na lista, sair da lista e usar a lista em
> http://www.mat.puc-rio.br/~nicolau/olimp/obm-l.html
> =
>


=
Instruções para entrar na lista, sair da lista e usar a lista em
http://www.mat.puc-rio.br/~nicolau/olimp/obm-l.html
=


[obm-l] Re: [obm-l] dúvida

2004-06-05 Por tôpico Fellipe Rossi



Entrada = 8 opções
Saida = 7 opções (não pode ser a de 
entrada)
 
A ordem importa, logo usamos o principio de 
contagem.
 
Total = 8*7 = 56 possibilidades.
 
 

  - Original Message - 
  From: 
  TSD 
  To: [EMAIL PROTECTED] 
  Sent: Sunday, June 06, 2004 12:56 
AM
  Subject: [obm-l] dúvida
  
  se uma sala tem 8 portas, então o número de 
  maneiras distintas de entrar e sair da mesma por uma porta diferente é 
  ?


Re: [obm-l] Matematica - Rato + Cubos - Prove

2004-06-03 Por tôpico Fellipe Rossi



Estive conversando com um amigo hoje e ele deu uma 
outra ideia pra fazer a questao
pinte cada "andar" do cubo como se fosse um 
tabuleiro de damas, de forma q cada cubo preto so esteja adjasscente face-a-face 
ocm um cubo branco
 
1o. andar:
X0X
0X0
X0X
 
2o. andar
0X0
X0X
0X0
 

3o. andar:
X0X
0X0
X0X
 
Assim sendo, ele terá que fazer sempre caminhos X 
-> 0 -> X -> 0 -> X -> 0...
Porém temos 14 X's e apenas 13 0's.
Como ele começa em X, ele certamente terminará em X 
(para que possa percorrer todos os cubos) porém o cubo que ele tem que terminar 
é o central do 2o andar que é 0.
Então é impossível! :)
 

  - Original Message - 
  From: 
  Fellipe Rossi 
  To: OBM 
  Sent: Thursday, June 03, 2004 11:15 
  AM
  Subject: Re: [obm-l] Matematica - Rato + 
  Cubos - Prove
  
  Vejam se esta solução é aceitável.
   
  Vamos considerar que A A' são as formas de se 
  movimentar para N e S, B e B'  Leste e Oeste e C e C' pra cima e pra 
  baixo
   
  Bom sao 27 cubos, então o rato tera que fazer 26 
  movimentos para passar apenas 1 vez em cada cubo.
  Porém sabemos que para que ele termine no meio 
  (aqui considerarei que ele está no cubo do centro da base, oq nao faz 
  diferença, pra construir o raciocínio melhor )
   
  Então ele terá que andar a mesma quantidade de 
  vezes pra B e B', assim como a msma quantidade pra A e A', porém ele 
  terá que andar C = C'+1 para terminar no meio.
   
  Sendo assim ele terá A+A'+B+B'+C+C' = 26, 
  como A = A' = n, B = B' = m temos que
  26 = 2n + 2m + C' + 1 + C' 
  ou seja, 26 = 2(n+m+C') + 1
   
  Desta forma, como n, m e C' são números Naturais, 
  temos que 2 (n+m+C') = PAR, logo 2 (n+m+C') + 1 é ÍMPAR.
   
  Então chegamos ao absurdo de que 26 é um número 
  IMPAR.
   
  Logo é IMPOSSÍVEL fazer o tal 
  caminho!!
   
  Espero que esteja certo! 
  Abraços do Rossi
  
- Original Message - 
From: 
Carlos Alberto 
To: [EMAIL PROTECTED] 
Sent: Thursday, June 03, 2004 9:48 
AM
Subject: [obm-l] Matematica - Rato + 
Cubos - Prove

Me passaram esse exercício, dizendo que era da Olimpiadas 
Capixaba.
 
Só que até então não consegui nenhuma solução!!!
 
Queria vê como seria a solução de vcs...
 
 
Havia sobre a mesa um cubo de queijo grande que foi partido em 27 
cubinhos pequenos de mesmo tamanho (3x3x3). O rato sai de sua toca 1x 
por dia para comer 1 unico cubo, de forma que, a cada cubinho q ele 
comia, no dia seguinte ele deveria comer um cubinho de face adjacente ao do 
dia anterior... Pede-se: É possível o rato comer o cubinho do centro no 
último dia??? Prove matematicamente 
 
 
Desde-já agradeço a todos.


Yahoo! 
Messenger - Fale com seus amigos online. Instale 
agora!


Re: [obm-l] Matematica - Rato + Cubos - Prove

2004-06-03 Por tôpico Fellipe Rossi




Vejam se esta solução é aceitável.
 
Vamos considerar que A A' são as formas de se 
movimentar para N e S, B e B'  Leste e Oeste e C e C' pra cima e pra 
baixo
 
Bom sao 27 cubos, então o rato tera que fazer 26 
movimentos para passar apenas 1 vez em cada cubo.
Porém sabemos que para que ele termine no meio 
(aqui considerarei que ele está no cubo do centro da base, oq nao faz diferença, 
pra construir o raciocínio melhor )
 
Então ele terá que andar a mesma quantidade de 
vezes pra B e B', assim como a msma quantidade pra A e A', porém ele 
terá que andar C = C'+1 para terminar no meio.
 
Sendo assim ele terá A+A'+B+B'+C+C' = 26, como 
A = A' = n, B = B' = m temos que
26 = 2n + 2m + C' + 1 + C' 
ou seja, 26 = 2(n+m+C') + 1
 
Desta forma, como n, m e C' são números Naturais, 
temos que 2 (n+m+C') = PAR, logo 2 (n+m+C') + 1 é ÍMPAR.
 
Então chegamos ao absurdo de que 26 é um número 
IMPAR.
 
Logo é IMPOSSÍVEL fazer o tal 
caminho!!
 
Espero que esteja certo! 
Abraços do Rossi

  - Original Message - 
  From: 
  Carlos 
  Alberto 
  To: [EMAIL PROTECTED] 
  Sent: Thursday, June 03, 2004 9:48 
  AM
  Subject: [obm-l] Matematica - Rato + 
  Cubos - Prove
  
  Me passaram esse exercício, dizendo que era da Olimpiadas Capixaba.
   
  Só que até então não consegui nenhuma solução!!!
   
  Queria vê como seria a solução de vcs...
   
   
  Havia sobre a mesa um cubo de queijo grande que foi partido em 27 
  cubinhos pequenos de mesmo tamanho (3x3x3). O rato sai de sua toca 1x por 
  dia para comer 1 unico cubo, de forma que, a cada cubinho q ele comia, no 
  dia seguinte ele deveria comer um cubinho de face adjacente ao do dia 
  anterior... Pede-se: É possível o rato comer o cubinho do centro no último 
  dia??? Prove matematicamente 
   
   
  Desde-já agradeço a todos.
  
  
  Yahoo! 
  Messenger - Fale com seus amigos online. Instale 
  agora!


Re: [obm-l] Matematica - Rato + Cubos - Prove

2004-06-03 Por tôpico Fellipe Rossi



Vejam se esta solução é aceitável.
 
Vamos considerar que A A' são as formas de se 
movimentar para N e S, B e B'  Leste e Oeste e C e C' pra cima e pra 
baixo
 
Bom sao 27 cubos, então o rato tera que fazer 26 
movimentos para passar apenas 1 vez em cada cubo.
Porém sabemos que para que ele termine no meio 
(aqui considerarei que ele está no cubo do centro da base, oq nao faz diferença, 
pra construir o raciocínio melhor )
 
Então ele terá que andar a mesma quantidade de 
vezes pra B e B', assim como a msma quantidade pra A e A', porém ele 
terá que andar C = C'+1 para terminar no meio.
 
Sendo assim ele terá A+A'+B+B'+C+C' = 26, como 
A = A' = n, B = B' = m temos que
26 = 2n + 2m + C' + 1 + C' 
ou seja, 26 = 2(n+m+C') + 1
 
Desta forma, como n, m e C' são números Naturais, 
temos que 2 (n+m+C') = PAR, logo 2 (n+m+C') + 1 é ÍMPAR.
 
Então chegamos ao absurdo de que 26 é um número 
IMPAR.
 
Logo é IMPOSSÍVEL fazer o tal 
caminho!!
 
Espero que esteja certo! 
Abraços do Rossi

  - Original Message - 
  From: 
  Carlos 
  Alberto 
  To: [EMAIL PROTECTED] 
  Sent: Thursday, June 03, 2004 9:48 
  AM
  Subject: [obm-l] Matematica - Rato + 
  Cubos - Prove
  
  Me passaram esse exercício, dizendo que era da Olimpiadas Capixaba.
   
  Só que até então não consegui nenhuma solução!!!
   
  Queria vê como seria a solução de vcs...
   
   
  Havia sobre a mesa um cubo de queijo grande que foi partido em 27 
  cubinhos pequenos de mesmo tamanho (3x3x3). O rato sai de sua toca 1x por 
  dia para comer 1 unico cubo, de forma que, a cada cubinho q ele comia, no 
  dia seguinte ele deveria comer um cubinho de face adjacente ao do dia 
  anterior... Pede-se: É possível o rato comer o cubinho do centro no último 
  dia??? Prove matematicamente 
   
   
  Desde-já agradeço a todos.
  
  
  Yahoo! 
  Messenger - Fale com seus amigos online. Instale 
  agora!


[obm-l] Re: [obm-l] livros para olimpíadas

2004-06-02 Por tôpico Fellipe Rossi



Eu sei que no IMPA vende.

  - Original Message - 
  From: 
  Victor 
  Machado 
  To: [EMAIL PROTECTED] 
  Sent: Tuesday, June 01, 2004 9:26 
PM
  Subject: [obm-l] livros para 
  olimpíadas
  
  Olá,
  aonde eu conseguiria aqueles livros citados no 
  sita da OBM; livros de problemas de olimpíadas passadas ?
   
  obrigado,
  victor


Re: [obm-l] LANCE INICIAL!

2004-06-01 Por tôpico Fellipe Rossi



Este problema ja foi respondido há decadas 
;)
 

  - Original Message - 
  From: 
  Johann Peter Gustav Lejeune 
  Dirichlet 
  To: [EMAIL PROTECTED] 
  Sent: Tuesday, June 01, 2004 3:05 
PM
  Subject: Re: [obm-l] LANCE INICIAL!
  
  Esse eu nao vou responder por completo (como sempre :) ) mas dou 
  umas dicas que deverao ser uteis.
  Pense de tras pra frente, voltando no tempo.Assim:
  Bem, supondo os dois amigos Arnaldo e Bernaldo bem 
  inteligentes,  faça o seguinte:
  Eles jogarao sem dar chance um ao outro de vencer.
  Se alguem, dogamos Arnaldo, fez 100, ele nao deu chance de Bernaldo 
  fazer 100.
  Pior, Arnaldo foi sadico o suficiente para que Bernaldo, querendo ou nao, 
  desse a vitoria para Arnaldo.
  Logo, no lance anterior Bernaldo deixou um numero de no maximo 99 e no 
  minimo 90 (senao Arnaldo nao teria total convicçao de que ganharia), e 
  Bernaldo somou um numero tal que a soma desse 100, e assim acabou 
  ganhando.
   
  Tente continuar daqui.
   
   
   
  [EMAIL PROTECTED] wrote:
  Turma! 
A discussão à respeito do probleminha dos monges foi perfeita e 
melhorainda foi a prova matemática proposta pelo Artur, que aliás, eu 
desconhecia.Ok!Dois amigos apostaram a conta do restaurante 
da seguinte maneira: um diziaqualquer número de 1 a 10 (inclusive 
ambos). Em seguida o outro somava a elequalquer número do mesmo 
intervalo. Depois o primeiro somava ao total outronúmero, sempre do 
mesmo intervalo, e assim alternadamente. O primeiro quechegasse 
exatamente ao número 100, valor da conta, ganharia a aposta. Quempoderia 
vencer com 
certeza?Abraços!__WebMail 
UNIFOR - 
http://www.unifor.br.=Instruções 
para entrar na lista, sair da lista e usar a lista 
emhttp://www.mat.puc-rio.br/~nicolau/olimp/obm-l.html=r/~nicolau/olimp/obm-l.html=
  
  TRANSIRE SVVM PECTVS MVNDOQVE POTIRI CONGREGATI EX TOTO ORBE 
  MATHEMATICI OB SCRIPTA INSIGNIA TRIBVERE Fields Medal(John Charles 
  Fields)
   
  N.F.C. (Ne Fronti Crede)
  
  
  Yahoo! 
  Mail - Participe da pesquisa global sobre o Yahoo! Mail. Clique 
  aqui!


Re: [obm-l] Geometria Espacial

2004-05-30 Por tôpico Fellipe Rossi



Eu também achei 3*pi*R^2 / 2
 
Achei q o lado da seção do cone é L=R*sqrt(3) 
(analisando a seçao meridional que será um triângulo equilátero inscrito em uma 
circunferência).
 
Então como a área lateral do cone será pi*L^2 / 2, 
cheguei a esse resultado.
 
Creio que se a resposta for mesmo ( pi R.R sqrt(3) 
) / 2  eu também erraria.
 
Abraços

  - Original Message - 
  From: 
  Fabio Contreiras 
  To: [EMAIL PROTECTED] 
  Sent: Sunday, May 30, 2004 2:23 PM
  Subject: [obm-l] Geometria Espacial
  
  Acabei de sair de uma prova no qual me deparei 
  com a questão :
   
  A área da superfície lateral de um cone 
  equilátero inscrito numa esfera de raio R é ?
   
   
  Gostaria de uma solução plausível para que o 
  resultado dê [ ( pi R.R sqrt(3) ) / 2 ] !
   
  ps. achei [ (  pi . 3.R.R ) / 2 
  ]
   
   
  obrigado!


[obm-l] Re: [obm-l] Análise I

2004-05-30 Por tôpico Fellipe Rossi



Caro Éder,
Muitos usuários desta lista possuem sistemas que 
não suportam certos tipos de símbolos.
Se puder, utilize apenas notações simples. 

P.ex: escreva 2 elevado a 3 como 2^3, 
etc...
Assim, todos poderão entender a questão! 
=)
 
Abraços

  - Original Message - 
  From: 
  Lista 
  OBM 
  To: [EMAIL PROTECTED] 
  Sent: Sunday, May 30, 2004 9:26 AM
  Subject: Re: [obm-l] Análise I
  
  Meu caro Morgado, de fato você tem razão, não fui claro na minha dúvida. 
  Vou tentar ser mais claro:
   
  i) Seja f: J --> R de classe C infinito no intervalo J. Suponha que 
  exista K > 0 t.q. |f(n)(x)| <= K para todo x em J e 
  todo n natural. Prove que, para x_o, x em J quaisquer vale f(x) = Somatório_[n 
  = 0...infinito]{[f(n)(x_o)]/n!}(x - x_o)^n.
   
  Início da solução: 
  chamei f(x) = p_n(x) + r_n(x), onde p_n é o polinômio de Taylor de ordem n de 
  f em torno de x_o e, pela Fórmula de Taylor com Resto de Lagrange, existe 
  c entre x e x_o t.q. r_n(x) =[f(n+1)(c).(x - x_o)]/(n+1)! , o 
  que implica que |r_n(x)| <= 
  [K|x-xo|n+1]/(n + 1)!. 
  Daí tenho que provar que  limn®¥ r_n(x) = 
  0. Augusto Cesar de Oliveira Morgado 
  <[EMAIL PROTECTED]> wrote:
  
Parece (os simbolos estao incompreensiveis) que se quer ptovar que o 
modulo de (x-a)^n / n! tende a 0 quando n tende a infinito. Pense nisso 
como o termo geral de uma serie, prove pelo criterio da razao de D'Alembert 
que ela eh convergente (a razao a(n+1)/a(n) tende a 0) e conclua que o termo 
geral tende a 0. 
== 
Mensagem  enviada  pelo  CIP  WebMAIL  - Nova 
Geração - v. 2.1 CentroIn Internet Provider         
 http://www.centroin.com.br Tel: (21) 2542-4849, (21) 
2295-3331        Fax: (21) 2295-2978 Empresa 100% 
Brasileira - Desde 1992 prestando servicos online -- 
Original Message --- From: Lista OBM 
<[EMAIL PROTECTED]> To: [EMAIL PROTECTED] Sent: Sat, 
29 May 2004 17:05:44 -0300 (ART) Subject: [obm-l] Análise I > 
Gostaria de saber se alguém pode me ajudar com os "dois problemas" 
abaixo: >   > > i) Sendo | r(x) | £ 
[K|x-xo|n+1]/(n 
+ 1)!, onde K > 0, prove que 
limn®¥ r(x) = 0; > 
  > ii) 
Seja  f: I à R de classe C2. Dado a em 
I, defina g: I ! à R por g(x) = 
[f(x) – f(a)]/(x – a) 
se x 
¹ a e g(a) = f´(a). 
Prove que g é de classe C1. Usando o pol. de Taylor com 
resto de Lagrange para f, cheguei que: 
limx®a g´(x) = 
[f´´(a)]/2 . Mas não estou conseguindo >  concluir que g é de classe 
C1. > $1  > Grato desde já com a possível ajuda 
de vocês. > > 

Yahoo! 
Messenger - Fale com seus amigos online. Instale 
agora! --- End of Original Message --- 

  
  
  Yahoo! 
  Messenger - Fale com seus amigos online. Instale 
  agora!


[obm-l] Re: [obm-l] Re: [obm-l]_Re:_[obm-l]_Re:_[obm-l]_OS_NÚMEROS_DO_ACASO!

2004-05-29 Por tôpico Fellipe Rossi
jogador 1  -  jogador 2
 0   2  =  1/6*1/6 = 1/36
 2   0 = 1/36
 1   1 = 1/36

somando = 3/36

Continuo dizendo
Me diga a diferença de se jogar 0 ou 4, ou 1 ou 5, no par ou impar?
Nada mais simples pra explicar as probabilidades iguais: o que importa eh
ser numero PAR ou IMPAR.


- Original Message -
From: "Chicao Valadares" <[EMAIL PROTECTED]>
To: <[EMAIL PROTECTED]>
Sent: Saturday, May 29, 2004 6:02 PM
Subject: Re: [obm-l] Re:
[obm-l]_Re:_[obm-l]_Re:_[obm-l]_OS_NÚMEROS_DO_ACASO!


> sendo a probabilidade 1/6 para cada numero como vc
> calculou a probabilidade da soma ser 2, por exemplo???
>
>
>
> > O seu raciocínio é incorreto por mais de um motivo.
> >
> > Se os dois jogadores escolherem com igual
> > probabilidade (1/6)
> > entre 0, 1, 2, 3, 4 e 5 então é verdade que a soma
> > pode
> > ser qualquer inteiro entre 0 e 10, mas eles não têm
> > igual
> > probabilidade, as probabilidades são dadas por:
> >
> >0 1 2 3 4 5 6 7 8
> > 9 10
> >   1/36  2/36  3/36  4/36  5/36  6/36  5/36  4/36
> > 3/36  2/36  1/36
> >
> > e se você somar as probabilidades para par e ímpar
> > vai ver que dá
> > exatamente 1/2 para cada um.
> >
> > Mais há um outro erro que torna o raciocínio acima
> > bastante irrelevante.
> > Digamos que pelas regras você só pudesse escolher um
> > número primo menor
> > do que 100. Se os dois jogadores escolherem um
> > número ao acaso com igual
> > probabilidade, quase certamente teremos
> > ímpar+ímpar=par. Mas se nós dois
> > jogarmos juntos, eu ganhar com ímpar e eu perceber
> > que você está jogando
> > desta maneira então eu passarei a jogar 2 todas as
> > vezes e portanto
> > passarei a ganhar quase todas as vezes. Ou seja:
> > escolher entre os números
> > com igual probabilidade é má estratégia e não se
> > deve supor que um bom
> > jogador proceda desta maneira.
> >
> > []s, N.
> >
> =
> > Instruções para entrar na lista, sair da lista e
> > usar a lista em
> > http://www.mat.puc-rio.br/~nicolau/olimp/obm-l.html
> >
> =
>
> =
> "O Binômio de Newton é tão belo como a Vênus de Milo.
> O que há é pouca gente para dar por isso... "
> Fernando Pessoa - Poesias de Alvaro Campos
>
> _
> As informações existentes nessa mensagem e no(s) arquivo(s) anexado(s)
> são
> para uso restrito, sendo seu sigilo protegido por lei. Caso não seja
> destinatário, saiba que leitura, divulgação ou cópia são proibidas.
> Favor
> apagar as informações e notificar o remetente. O uso impróprio será
> tratado
> conforme as normas da empresa e a legislação em vigor. Agradecemos sua
> colaboração.
>
>
> The information mentioned in this message and in the archives attached
> are
> of restricted use, and its privacy is protected by law. If you are not
> the
> addressee, be aware that reading, disclosure or copy are forbidden.
> Please
> delete this information and notify the sender. Inappropriate use will
> be
> tracted according to company's rules and valid laws. Thank you for your
> cooperation.
>
> __
>
> Participe da pesquisa global sobre o Yahoo! Mail:
> http://br.surveys.yahoo.com/global_mail_survey_br
> =
> Instruções para entrar na lista, sair da lista e usar a lista em
> http://www.mat.puc-rio.br/~nicolau/olimp/obm-l.html
> =
>
>


=
Instruções para entrar na lista, sair da lista e usar a lista em
http://www.mat.puc-rio.br/~nicolau/olimp/obm-l.html
=


[obm-l] Re: [obm-l] Re: [obm-l]_Re:_[obm-l]_OS_NÚMEROS_DO_ACASO!

2004-05-27 Por tôpico Fellipe Rossi
Certamente não
Ou você joga par ou impar colocando 10, 7, esses numeros?? eu só uso 0 ou
1...
no caso temos 4 possibilidades
P + P = P
P + I = I
I + P = I
I + I = P

Se considerarmos que a probabilidade de uma pessoa colocar P seja igual a I,
temos 1/4 de chances pra cada caso.
Meio a meio.

- Original Message -
From: "Chicao Valadares" <[EMAIL PROTECTED]>
To: <[EMAIL PROTECTED]>
Sent: Thursday, May 27, 2004 3:23 PM
Subject: Re: [obm-l] Re: [obm-l]_Re:_[obm-l]_OS_NÚMEROS_DO_ACASO!


> Se este for o simples par ou impar que eu conheço,cada
> um colocava sua mao na frente(uma mao)e contava-se a
> soma dos dedos das duas maos.Quem tivesse escolhido
> par e a soma tivesse dado par, ganhava.Bem , para mim
> era obvio que quem escolhesse par tinha mais chances,
> simplesmente pq o zero era possivel e o resultado que
> interessava era a soma, portanto havia 11 numeros
> possiveis para a soma = 0,1,2,3,4,5,6,7,8,9,10 e
> desses 6 eram par e 5 eram impares.A coisa ficava
> igual se tirassemos o zero.
>
> []´s
>
>
>  --- "Nicolau C. Saldanha" <[EMAIL PROTECTED]>
> escreveu: > On Tue, May 25, 2004 at 11:18:00PM -0300,
> Fellipe
> > Rossi wrote:
> > > "PASMEM! O jogo do par ou ímpar é, sem sombra de
> > dúvidas, favorável a C.
> > > Abraços!"
> > >
> > > Por quê?
> > >
> > > Rossi
> >
> > Talvez outros adivinhem melhor do que eu, mas eu não
> > tenho a menor idéia
> > de quem o o que seja este "C." que estaria sendo
> > favorecido.
> >
> > Meu melhor palpite é que estamos falando daquele
> > raciocínio
> > (completamente errado) que diz que PAR tem
> > probabilidade maior
> > do que ÍMPAR de ganhar, pois para PAR há duas
> > possibilidades
> > (os dois jogadores jogam números pares; os dois
> > jogadores
> > jogam números ímpares) enquenato para ÍMPAR só há
> > uma possibilidade
> > (os dois jogadores escolhem números de paridades
> > diferentes).
> >
> > []s, N.
> >
> =
> > Instruções para entrar na lista, sair da lista e
> > usar a lista em
> > http://www.mat.puc-rio.br/~nicolau/olimp/obm-l.html
> >
>
=r/~
nicolau/olimp/obm-l.html
> >
> =
>
> =
> "O Binômio de Newton é tão belo como a Vênus de Milo.
> O que há é pouca gente para dar por isso... "
> Fernando Pessoa - Poesias de Alvaro Campos
>
> _
> As informações existentes nessa mensagem e no(s) arquivo(s) anexado(s)
> são
> para uso restrito, sendo seu sigilo protegido por lei. Caso não seja
> destinatário, saiba que leitura, divulgação ou cópia são proibidas.
> Favor
> apagar as informações e notificar o remetente. O uso impróprio será
> tratado
> conforme as normas da empresa e a legislação em vigor. Agradecemos sua
> colaboração.
>
>
> The information mentioned in this message and in the archives attached
> are
> of restricted use, and its privacy is protected by law. If you are not
> the
> addressee, be aware that reading, disclosure or copy are forbidden.
> Please
> delete this information and notify the sender. Inappropriate use will
> be
> tracted according to company's rules and valid laws. Thank you for your
> cooperation.
>
> __
>
> Participe da pesquisa global sobre o Yahoo! Mail:
> http://br.surveys.yahoo.com/global_mail_survey_br
> =
> Instruções para entrar na lista, sair da lista e usar a lista em
> http://www.mat.puc-rio.br/~nicolau/olimp/obm-l.html
> =
>


=
Instruções para entrar na lista, sair da lista e usar a lista em
http://www.mat.puc-rio.br/~nicolau/olimp/obm-l.html
=


[obm-l] Re: [obm-l] Re: [obm-l] Re: [obm-l] OS NÚMEROS DO ACASO!

2004-05-26 Por tôpico Fellipe Rossi



Eis o problema.
 
Poderia explicar o por quê, Jorge Luís (ou alguém 
que saiba! =D )
 
Abraços, Rossi
 
 
- Original Message -From: <[EMAIL PROTECTED]>To: 
<[EMAIL PROTECTED]>Sent: 
Thursday, May 20, 2004 8:28 PMSubject: [obm-l] RETIFICAÇÃO!
Dois jogadores, L e C, mostram, simultâneamente, 2 
ou 3 dedos. Se a somade dedos mostrados é par, então L ganha tal soma de C; 
se a soma é impar,então L  perde esta soma para C. A quem o jogo é 
favorável?
 
 

- Original Message - 
From: <[EMAIL PROTECTED]>
To: <[EMAIL PROTECTED]>
Sent: Tuesday, May 25, 2004 7:19 PM
Subject: [obm-l] OS NÚMEROS DO ACASO!
 
PASMEM! O jogo do par ou ímpar é, sem sombra de 
dúvidas, favorável a C. Abraços!


[obm-l] Re: [obm-l] OS NÚMEROS DO ACASO!

2004-05-25 Por tôpico Fellipe Rossi



"PASMEM! O jogo do par ou ímpar 
é, sem sombra de dúvidas, favorável a C. Abraços!"
 
Por quê?
 
Rossi


Re: [obm-l] En:colegio naval

2004-05-24 Por tôpico Fellipe Rossi
Rogério quando vc colocou o 13/2 em evidência, vc dividiu os 2 numeros do
produto do denominador por 2. Porém ali há uma multiplicação
ou seja, quando aplicares a distributiva, não ficaremos com o valor original

Por exemplo, (13/2)*(1/1*2) = 13/4 e não 13/2*4 como diz no enunciado.

Abraços!
- Original Message -
From: "Rogério Possi Júnior" <[EMAIL PROTECTED]>
To: <[EMAIL PROTECTED]>
Sent: Monday, May 24, 2004 10:52 AM
Subject: RE: [obm-l] En:colegio naval


> Caro Leandro,
>
> Acho que a questão pode ser assim resolvida:
>
> Seja S=13/2.4 + 13/4.6 + ... + 13/50.52
>
> -> S=(13/2).{ 1/1.2 + 1/2.3 + ... + 1/25.26 }
>
> Ora, a soma entre chaves pode ser assim representada:
>
> 1/1.2 = 1-1/2
> 1/2.3 = 1/2 - 1/3
>
>   ...   ...
>
> 1/25.26 = 1/25 - 1/26
>
> Somando as parcelas chegamos em
>
> 1/1.2 + 1/2.3 + ... + 1/25.26 = 1 - 1/26 = 25/26
>
> -> S=13/2 . 25/26 = 25/4 (RESPOSTA D)
>
> Espero ter ajudado.
>
> Abraços, Rogério
>
>
>
> >From: "leandro-epcar" <[EMAIL PROTECTED]>
> >Reply-To: [EMAIL PROTECTED]
> >To: "obm-l" <[EMAIL PROTECTED]>
> >Subject: [obm-l] En:colegio naval
> >Date: Mon, 24 May 2004 10:22:11 -0300
> >
> >-- Início da mensagem original ---
> >
> >   De: "leandro-epcar" leandro-
> >[EMAIL PROTECTED]
> > Para: "obm-l" [EMAIL PROTECTED]
> >   Cc:
> > Data: Mon, 24 May 2004 10:08:21 -0300
> >  Assunto: colegio naval
> >
> >   Alguem poderia me dar uma ideia nesta questao ,nao
> >consigo achar uma sequencia ,nem mesmo calcular, esta
> >questao.
> >grato
> >leandro
> >
> >
> >
> >Colegio naval   1994
> >
> >   Sabendo-se que a seguinte identidade  (AX + BY)/XY =
> >A/Y + B/X é verdadeira para quaisquer números reais A,B,
> >X<>0,Y<>0,
> >   o valor de 13/(2*4)+ 13 /(4*6)+ 13/ ( 6*8) +...+13/
> >(50*52)
> >
> >(A)25/16
> >(B)25/12
> >(C)25/8
> >(D)25/4
> >(E)25/2
> >
> >
> >
> >__
> >Acabe com aquelas janelinhas que pulam na sua tela.
> >AntiPop-up UOL - É grátis!
> >http://antipopup.uol.com.br/
> >
> >
> >
>
>__
> >Acabe com aquelas janelinhas que pulam na sua tela.
> >AntiPop-up UOL - É grátis!
> >http://antipopup.uol.com.br/
> >
> >
> >
> >=
> >Instruções para entrar na lista, sair da lista e usar a lista em
> >http://www.mat.puc-rio.br/~nicolau/olimp/obm-l.html
> >=
>
> _
> MSN Messenger: instale grátis e converse com seus amigos.
> http://messenger.msn.com.br
>
> =
> Instruções para entrar na lista, sair da lista e usar a lista em
> http://www.mat.puc-rio.br/~nicolau/olimp/obm-l.html
> =
>
>


=
Instruções para entrar na lista, sair da lista e usar a lista em
http://www.mat.puc-rio.br/~nicolau/olimp/obm-l.html
=


Re: [obm-l] %

2004-05-23 Por tôpico Fellipe Rossi



supondo o valor inicial = x
aumento percentual de i = x(1+i)
 
=> em 1 mes, temos 
x*(1+0,2) = 1,2x
 
=> em 2 meses, temos 
1,2x*1,12 = 1,344x
ou seja, o aumento acumulado foi de 
34,4%
 
Rossi
- Original Message - 
From: "elton francisco ferreira" <[EMAIL PROTECTED]>
To: <[EMAIL PROTECTED]>
Sent: Sunday, May 23, 2004 2:08 PM
Subject: [obm-l] %
> O valor de uma mercadoria em reais, subiu 20% no mês e> 12% 
no mês seguinte. No intervalo desses dois meses, a> mercadoria 
subiu:> > 32%> 32.4%> 34.4%> 34%> 33% 
> > 
__> 
> Yahoo! Messenger - Fale com seus amigos online. Instale agora! > 
http://br.download.yahoo.com/messenger/> 
=> 
Instruções para entrar na lista, sair da lista e usar a lista em> 
http://www.mat.puc-rio.br/~nicolau/olimp/obm-l.html> 
=> 
> 


[obm-l] Re: [obm-l] aritmética I

2004-05-22 Por tôpico Fellipe Rossi
a q.1 está esquisita.

2)
 x+y = 329
maior resto possivel numa divisão por y = y-1
Considerando x>y, x=13y+(y-1)
=>14y-1+y=329
=>15y=330,
Então y=22.
Logo, x=307.

3)
Supondo que o número é x.
Acrescentando 14 a sua direita, temos que o numero aumentara de 100 vezes +
14
ou seja, temos 100x+14 = x+2093
=> 99x=2079
=> x=21

Abraços

- Original Message -
From: "elton francisco ferreira" <[EMAIL PROTECTED]>
To: <[EMAIL PROTECTED]>
Sent: Saturday, May 22, 2004 7:00 PM
Subject: [obm-l] aritmética I


> 1 - Numa divisão, o dividendo e o quociente, 8.
> Encontre o divisor, sabendo que o resto é o maior
> possível.
>
>
>
>
> 2 - A soma de dois números é 329. A divisão do maior
> pelo menor dá quociente 13 e o resto o mairo possível.
> Quais são esses números?
>
> 3 - Qual o número fica aumentado em 2093 unidades
> quando acrenta-se a sua direita o número 14?
>
> __
>
> Yahoo! Messenger - Fale com seus amigos online. Instale agora!
> http://br.download.yahoo.com/messenger/
> =
> Instruções para entrar na lista, sair da lista e usar a lista em
> http://www.mat.puc-rio.br/~nicolau/olimp/obm-l.html
> =
>
>


=
Instruções para entrar na lista, sair da lista e usar a lista em
http://www.mat.puc-rio.br/~nicolau/olimp/obm-l.html
=


Re: [obm-l] PARADOXO DO PRISIONEIRO!

2004-05-22 Por tôpico Fellipe Rossi
Eu penso que será 1/2
Mas se fosse 1/2 essa questão não seria proposta (trivial) heheheh
então creio que estou errado, mas nao sei o por quê...


PS: aquilo que eu disse sobre o "par ou impar" ta errado?

Abraços
- Original Message -
From: <[EMAIL PROTECTED]>
To: <[EMAIL PROTECTED]>
Sent: Friday, May 21, 2004 8:27 PM
Subject: [obm-l] PARADOXO DO PRISIONEIRO!


> Ok! Rogério e demais colegas! Sua elucidação à respeito do "lance inicial"
foi o
> "tiro de misericórdia". Agora, por enquanto, o enigma do "par ou ímpar"
está
> indecifrável, mas aguardem que, em breve, teremos surpresas. Ok!
Abraços!!
>
>
> Em uma prisão há três detentos. O carcereiro os informa que somente um
será
> libertado e que os outros dois serão fuzilados, mas não pode revelar os
nomes.
> O prisioneiro A sabe então que sua chance de sobreviver é de 1/3. Para
> descobrir mais ele pergunta em segredo ao carcereiro o nome de apenas um
dos
> prisioneiros que será executado. O carcereiro diz "B". Com essa
informação,
> qual a nova probabilidade de A sobreviver?
>
>
>
> Bom Final de Semana!
>
>
>
> __
> WebMail UNIFOR - http://www.unifor.br.
> =
> Instruções para entrar na lista, sair da lista e usar a lista em
> http://www.mat.puc-rio.br/~nicolau/olimp/obm-l.html
> =
>
>


=
Instruções para entrar na lista, sair da lista e usar a lista em
http://www.mat.puc-rio.br/~nicolau/olimp/obm-l.html
=


[obm-l] Re: [obm-l] Múltiplo

2004-05-21 Por tôpico Fellipe Rossi
multiplo de 3 é o numero cuja soma dos algarismos seja multipla de tres
por exemplo
12345 é multiplo de três pois 1+2+3+4+5 = 15, que é multiplo de 3.

- Original Message -
From: "Junior" <[EMAIL PROTECTED]>
To: <[EMAIL PROTECTED]>
Sent: Friday, May 21, 2004 8:36 AM
Subject: [obm-l] Múltiplo


> Dado o problema:
>
> São formados numeros de 3 algarismos distintos usando-se os dígitos
1,2,3,6 e 7
>
> Depois de formados, um desses numeros (de 3 algarismos) foi sorteado. Qual
a
> probabilidade dele ser um multiplo de 3?
>
> ->   Encontrei o espaço amostral A(5,3)
> -> Mas não consigo observar como ser multiplo de 3?
>
> Alguem poderia me ajudar.
>
> Desde ja agradeço.
> =
> Instruções para entrar na lista, sair da lista e usar a lista em
> http://www.mat.puc-rio.br/~nicolau/olimp/obm-l.html
> =
>
>


=
Instruções para entrar na lista, sair da lista e usar a lista em
http://www.mat.puc-rio.br/~nicolau/olimp/obm-l.html
=


[obm-l] Re: [obm-l] RETIFICAÇÃO!

2004-05-20 Por tôpico Fellipe Rossi
Para C, tanto faz sair 2-3 ou 3-2 por isso ele jogaria qualquer um.

Bom eu diria que se eu fosse o jogador L eu jogaria apenas 3
pois com 2 ou 3 as chances de eu ganhar seriam as mesmas, então eu jogaria 3
para que o meu prêmio fosse maior

Por isso eu acho que o jogo seria mais favorável para L.


- Original Message -
From: <[EMAIL PROTECTED]>
To: <[EMAIL PROTECTED]>
Sent: Thursday, May 20, 2004 8:28 PM
Subject: [obm-l] RETIFICAÇÃO!


> Oi, Pessoal! Cometi um pequeno erro de grafia na palavra assistente, mas
> aproveitando a carona, vamos a uma disputa de par ou ímpar no problema
abaixo:
>
>
> Dois jogadores, L e C, mostram, simultâneamente, 2 ou 3 dedos. Se a soma
de
> dedos mostrados é par, então L ganha tal soma de C; se a soma é impar,
então L
> perde esta soma para C. A quem o jogo é favorável?
>
>
>
> Divirtam-se!
>
> __
> WebMail UNIFOR - http://www.unifor.br.
> =
> Instruções para entrar na lista, sair da lista e usar a lista em
> http://www.mat.puc-rio.br/~nicolau/olimp/obm-l.html
> =
>
>


=
Instruções para entrar na lista, sair da lista e usar a lista em
http://www.mat.puc-rio.br/~nicolau/olimp/obm-l.html
=


[obm-l] Re: [obm-l] RETIFICAÇÃO!

2004-05-20 Por tôpico Fellipe Rossi
Jogam aleatoriamente?


- Original Message -
From: <[EMAIL PROTECTED]>
To: <[EMAIL PROTECTED]>
Sent: Thursday, May 20, 2004 8:28 PM
Subject: [obm-l] RETIFICAÇÃO!


> Oi, Pessoal! Cometi um pequeno erro de grafia na palavra assistente, mas
> aproveitando a carona, vamos a uma disputa de par ou ímpar no problema
abaixo:
>
>
> Dois jogadores, L e C, mostram, simultâneamente, 2 ou 3 dedos. Se a soma
de
> dedos mostrados é par, então L ganha tal soma de C; se a soma é impar,
então L
> perde esta soma para C. A quem o jogo é favorável?
>
>
>
> Divirtam-se!
>
> __
> WebMail UNIFOR - http://www.unifor.br.
> =
> Instruções para entrar na lista, sair da lista e usar a lista em
> http://www.mat.puc-rio.br/~nicolau/olimp/obm-l.html
> =
>
>


=
Instruções para entrar na lista, sair da lista e usar a lista em
http://www.mat.puc-rio.br/~nicolau/olimp/obm-l.html
=


Re: [obm-l] + funcoes

2004-05-17 Por tôpico Fellipe Rossi



A receita será o preço vezes o numero de 
frequentadores
ou seja,
 
Receita = R(x)
R(x) = p*x => R(x)=(100-0,2x)x = -0,2x^2 + 
100x
 
como p=60, temos que 100-0,2x=60
logo, x=200 pessoas
logo a receita será p*x = 60*200 = 
R$12.000,00
 
R(x) é uma função do 2o.grau como o coeficiente de 
x^2 eh negativo, tem concavidade para baixo.
Logo vai ter um valor máximo no 
vértice
Porém queremos o valor de x pra termos receita 
máxima, então precisamos do x do vértice (x_v)
x_v = -b/2a = 100/0,2*2 = 250 pessoas.
Logo p=100-0,2x = 100-50 = R$ 
50,00
 
 
- Original Message - 
From: "aryqueirozq" <[EMAIL PROTECTED]>
To: <[EMAIL PROTECTED]>
Sent: Monday, May 17, 2004 10:28 PM
Subject: [obm-l] + funcoes
> > >  Me desculpem pelas perguntas, mas por que 
ainda estou > na 8ª série.> > >  O preço de 
ingresso numa peça de teatro (p) relaciona-> se com a quantidade de 
frequentadores (x) por sessão > através da relação;> > p = 
- 0,2x + 100> > a) Qual a receita arrecadada por sessão, se o 
preço de > ingresso for R$60,00?> b) Qual o preço que deve ser 
cobrado para dar a máxima > receita por sessão?


Re: [obm-l] + funcoes

2004-05-17 Por tôpico Fellipe Rossi
Desculpe quando mandei a msg nao tinha chegado esta ainda...

- Original Message -
From: "Augusto Cesar de Oliveira Morgado" <[EMAIL PROTECTED]>
To: <[EMAIL PROTECTED]>
Sent: Monday, May 17, 2004 11:41 PM
Subject: Re: [obm-l] + funcoes


> A receita vale R = px = -0,2(x^2)+ 100x.
> a) Se p = 60, x=200 e R= 12 000
> b) R sera maximo se x = -100/2(-0,2) = 250 e p=50.
>
> ==
> Mensagem  enviada  pelo  CIP  WebMAIL  - Nova Geração - v. 2.1
> CentroIn Internet Provider  http://www.centroin.com.br
> Tel: (21) 2542-4849, (21) 2295-3331Fax: (21) 2295-2978
> Empresa 100% Brasileira - Desde 1992 prestando servicos online
>
>
> -- Original Message ---
> From: "aryqueirozq" <[EMAIL PROTECTED]>
> To: "[EMAIL PROTECTED]" <[EMAIL PROTECTED]>
> Sent: Mon, 17 May 2004 22:28:43 -0300
> Subject: [obm-l] + funcoes
>
> > Me desculpem pelas perguntas, mas por que ainda estou
> > na 8ª série.
> >
> >  O preço de ingresso numa peça de teatro (p) relaciona-
> > se com a quantidade de frequentadores (x) por sessão
> > através da relação;
> >
> > p = - 0,2x + 100
> >
> > a) Qual a receita arrecadada por sessão, se o preço de
> > ingresso for R$60,00?
> > b) Qual o preço que deve ser cobrado para dar a máxima
> > receita por sessão?
> >
> >  agradeço
> >
> >
__
> > Acabe com aquelas janelinhas que pulam na sua tela.
> > AntiPop-up UOL - É grátis!
> > http://antipopup.uol.com.br/
> >
> >
=
> > Instruções para entrar na lista, sair da lista e usar a lista em
> > http://www.mat.puc-rio.br/~nicolau/olimp/obm-l.html
> >
=
> --- End of Original Message ---
>
> =
> Instruções para entrar na lista, sair da lista e usar a lista em
> http://www.mat.puc-rio.br/~nicolau/olimp/obm-l.html
> =
>
>


=
Instruções para entrar na lista, sair da lista e usar a lista em
http://www.mat.puc-rio.br/~nicolau/olimp/obm-l.html
=


Re: [obm-l] Algoritmo

2004-05-13 Por tôpico Fellipe Rossi
crie uma array M de 1..10
a=0
i=0
n=9

repeat

n=n-1

repeat
if m[i]>m[i+1] then
a=m[i+1];
m[i+1]=m[i];
m[1]=a;

( o procedimento do 'a' é pra vc nao perder o valor do m[i+1] )
(cada loop desses vai faze com q o menos seja jogado pro ultimo)

i=i+1 (incremento)
until i=n

until n=0

não me lembro das linguagens hehehe
eu misturo mto pascal com C entao ve se vc consegue sair com isso :)

[]'s


- Original Message -
From: "rickufrj" <[EMAIL PROTECTED]>
To: "obm-l" <[EMAIL PROTECTED]>
Sent: Thursday, May 13, 2004 11:20 PM
Subject: [obm-l] Algoritmo


> Olá pessoal , será que alguém poderia me ajudar nesse
> algoritmo :
>
> Fazer um algoritmo que leia a media de 10 alunos e em
> seguida as coloque em ordem decrescente.
>
> Eu to me enrolando um pouco , pois não consigo montar
> um algoritmo compacto ,nas idéias que eu tive , estava
> descambando para um lado muito grande  .
> Será que alguém poderia me ajudar utilizando os loop's
> for ou outro tipo de loop ?
> Agradeço a atenção
> Abraços
> Luiz H. Barbosa
>
> __
> Acabe com aquelas janelinhas que pulam na sua tela.
> AntiPop-up UOL - É grátis!
> http://antipopup.uol.com.br/
>
>
>
> =
> Instruções para entrar na lista, sair da lista e usar a lista em
> http://www.mat.puc-rio.br/~nicolau/olimp/obm-l.html
> =
>
>


=
Instruções para entrar na lista, sair da lista e usar a lista em
http://www.mat.puc-rio.br/~nicolau/olimp/obm-l.html
=


[obm-l] Mediatrizes

2004-05-12 Por tôpico Fellipe Rossi



Tome no plano xOy, A(a,0) e B(o,b), a e b ReaisSejam A'(a+d,0) e B' 
(0,b+d), d Real.Mostre que todas as mediatrizes dos segmentos A'B' Se 
interceptam em umponto.
 
Alguem resolveu este?
Abraços


Re: [obm-l] Demonstracoes no ensino medio

2004-05-12 Por tôpico Fellipe Rossi
Certamente não são todos os cursos.
O que eu quis dizer é que quando um curso bem sucedido recebe matrícula de
muitos alunos novos que, em sua maioria, não tem uma base forte, não há
tempo (quase sempre 1 ano, ou seja curtíssimo prazo) de ensinar toda a
matéria nos seus minimos detalhes, com demonstrações, etc. (isso me
referindo a um pré-vestibular "normal")

Então obtém amior sucesso aquele curso que consegue fazer a "bitolação mais
produtiva".

E o pior! A felicidade do aluno está em tirar a nota mais alta e não em
aprender!

PS: As cotas estão se alastrando! Minha opinião é ser totalmente contra
elas! Estudo da UERJ, faço Licensiatura em Matemática e, pasmem, outro dia o
profesor de cálculo teve que parar a aula pra ensinar a um grupo como se
fatorava uma equação do 2o. grau!! (BÁSKARA!)

Não quero começar a discussão sobre as cotas. Acho que estamos ficando bem
off-topic mesmo.

Abraços

- Original Message -
From: "Nicolau C. Saldanha" <[EMAIL PROTECTED]>
To: <[EMAIL PROTECTED]>
Sent: Wednesday, May 12, 2004 12:00 PM
Subject: Re: [obm-l] Demonstracoes no ensino medio


> On Wed, May 12, 2004 at 10:26:25AM -0300, Fellipe Rossi wrote:
> > A verdade é que o que vende livros e conquista novos alunos não é o
> > crescimento acadêmico, são os resultados.
> > As pessoas escolhem fazer esse ou aquele curso ou colégio porque "ele
aprova
> > mais do q os outros no vestibular". E os cursos bitolam os alunos com
> > fórmulas e teoremas que estes nem sabem de onde surgiram, mas que dão
> > resultados num concurso.
>
> Estamos ficando um pouco off-topic, mas eu discordo das generalizações
> feitas pelo Fellipe. Alguns cursos talvez "bitolem" os alunos,
> mas eu conheço cursos que não têm em absoluto este procedimento.
> Isto de sair decorando fórmulas só dá resultados a curtíssimo prazo
> e mesmo assim só se o concurso for muito mal feito. Acho que é bem
> claro que quando a aluno decora sem entender ele esquece logo e mesmo
> enquanto lembra do que decorou ele só sabe fazer exatamente aquele
problema
> decorado; qualquer mudança, por menor que seja, já desmonta o aluno.
> Finalmente, eu acho que a existência de uma competição como o vestibular
> ajuda a melhorar o ensino: não existindo esta pressão, mais escolas
> e mais alunos se acomodariam na mediocridade.
>
> []s, N.
> =
> Instruções para entrar na lista, sair da lista e usar a lista em
> http://www.mat.puc-rio.br/~nicolau/olimp/obm-l.html
> =
>
>


=
Instruções para entrar na lista, sair da lista e usar a lista em
http://www.mat.puc-rio.br/~nicolau/olimp/obm-l.html
=


Re: [obm-l] Demonstracoes no ensino medio

2004-05-12 Por tôpico Fellipe Rossi
A maioria ?

- Original Message -
From: "Augusto Cesar de Oliveira Morgado" <[EMAIL PROTECTED]>
To: <[EMAIL PROTECTED]>
Sent: Wednesday, May 12, 2004 12:03 PM
Subject: Re: [obm-l] Demonstracoes no ensino medio


> Perdão, dão resultados em concursos mal feitos.
>
> ==
> Mensagem  enviada  pelo  CIP  WebMAIL  - Nova Geração - v. 2.1
> CentroIn Internet Provider  http://www.centroin.com.br
> Tel: (21) 2542-4849, (21) 2295-3331Fax: (21) 2295-2978
> Empresa 100% Brasileira - Desde 1992 prestando servicos online
>
>
> -- Original Message ---
> From: "Fellipe Rossi" <[EMAIL PROTECTED]>
> To: <[EMAIL PROTECTED]>
> Sent: Wed, 12 May 2004 10:26:25 -0300
> Subject: Re: [obm-l] Demonstracoes no ensino medio
>
> > A verdade é que o que vende livros e conquista novos alunos não é o
> > crescimento acadêmico, são os resultados.
> > As pessoas escolhem fazer esse ou aquele curso ou colégio porque
> > "ele aprova mais do q os outros no vestibular". E os cursos bitolam
> > os alunos com fórmulas e teoremas que estes nem sabem de onde
> > surgiram, mas que dão resultados num concurso.
> >
> > - Original Message -
> > From: "Augusto Cesar de Oliveira Morgado" <[EMAIL PROTECTED]>
> > To: <[EMAIL PROTECTED]>
> > Sent: Wednesday, May 12, 2004 7:55 AM
> > Subject: Re: [obm-l] Demonstracoes no ensino medio
> >
> > > E eu fico mais abismado ainda, porque, como dizem alguns "colegas" e
> > alguns
> > > autores de livros didáticos, a ausencia das demonstrações é para
tornar o
> > > ensino objetivo e prático.
> > >
> > >
> > >
> > > -- Original Message ---
> > > From: "Marcelo Salhab Brogliato" <[EMAIL PROTECTED]>
> > > To: <[EMAIL PROTECTED]>
> > > Sent: Wed, 12 May 2004 03:18:08 -0300
> > > Subject: Re: [obm-l] Demonstracoes no ensino medio
> > >
> > > > Claudio,
> > > > nao participo muito da lista pois pouco sei sobre os assuntos que
voces
> > > > falam. Mas fiz ETEP, um dos colegios tecnicos mais conceituados da
minha
> > > > regiao (SJCampos-SP) e nunca tive demonstracao nenhuma. Alias, era
> > decorar
> > > > formula e aplicar em provas. Agora que estou estudando para ITA/IME
e
> > > > fazendo cursinho no Poliedro, estou tendo todas as demonstracoes e
> > ficando
> > > > abismado como as coisas sao simples.
> > > > Infelizmente vejo que nenhum de meus amigos/conhecidos jamais viu
uma
> > > > demonstração.
> > > >
> > > > Marcelo
> > >
> > >
=
> > > Instruções para entrar na lista, sair da lista e usar a lista em
> > > http://www.mat.puc-rio.br/~nicolau/olimp/obm-l.html
> > >
=
> > >
> >
> >
=
> > Instruções para entrar na lista, sair da lista e usar a lista em
> > http://www.mat.puc-rio.br/~nicolau/olimp/obm-l.html
> >
=
> --- End of Original Message ---
>
> =
> Instruções para entrar na lista, sair da lista e usar a lista em
> http://www.mat.puc-rio.br/~nicolau/olimp/obm-l.html
> =
>
>


=
Instruções para entrar na lista, sair da lista e usar a lista em
http://www.mat.puc-rio.br/~nicolau/olimp/obm-l.html
=


Re: [obm-l] Demonstracoes no ensino medio

2004-05-12 Por tôpico Fellipe Rossi
A verdade é que o que vende livros e conquista novos alunos não é o
crescimento acadêmico, são os resultados.
As pessoas escolhem fazer esse ou aquele curso ou colégio porque "ele aprova
mais do q os outros no vestibular". E os cursos bitolam os alunos com
fórmulas e teoremas que estes nem sabem de onde surgiram, mas que dão
resultados num concurso.


- Original Message -
From: "Augusto Cesar de Oliveira Morgado" <[EMAIL PROTECTED]>
To: <[EMAIL PROTECTED]>
Sent: Wednesday, May 12, 2004 7:55 AM
Subject: Re: [obm-l] Demonstracoes no ensino medio


> E eu fico mais abismado ainda, porque, como dizem alguns "colegas" e
alguns
> autores de livros didáticos, a ausencia das demonstrações é para tornar o
> ensino objetivo e prático.
>
>
>
> -- Original Message ---
> From: "Marcelo Salhab Brogliato" <[EMAIL PROTECTED]>
> To: <[EMAIL PROTECTED]>
> Sent: Wed, 12 May 2004 03:18:08 -0300
> Subject: Re: [obm-l] Demonstracoes no ensino medio
>
> > Claudio,
> > nao participo muito da lista pois pouco sei sobre os assuntos que voces
> > falam. Mas fiz ETEP, um dos colegios tecnicos mais conceituados da minha
> > regiao (SJCampos-SP) e nunca tive demonstracao nenhuma. Alias, era
decorar
> > formula e aplicar em provas. Agora que estou estudando para ITA/IME e
> > fazendo cursinho no Poliedro, estou tendo todas as demonstracoes e
ficando
> > abismado como as coisas sao simples.
> > Infelizmente vejo que nenhum de meus amigos/conhecidos jamais viu uma
> > demonstração.
> >
> > Marcelo
>
> =
> Instruções para entrar na lista, sair da lista e usar a lista em
> http://www.mat.puc-rio.br/~nicolau/olimp/obm-l.html
> =
>


=
Instruções para entrar na lista, sair da lista e usar a lista em
http://www.mat.puc-rio.br/~nicolau/olimp/obm-l.html
=


Re: [obm-l] Duvidas

2004-05-11 Por tôpico Fellipe Rossi
Para a primeira questão, considere que a equação geral da parábola é:
y=ax^2+bx+c
substitua os 3 pontos dados e vc vai cair em um sistema linear de 3
incógnitas.
Assim, vai cahar a equação da parábola e basta substituir x por 2,5

Para o segundo problema, note que se a parábola é simétrica em relação a uma
função constante, o valoe de a é o mesmo em módulo porém com sinal trocado
(o que faz com que a concavidade seja "pra cima")
Como a equação -x^2+2 é simétrica em relação ao eixo y (raízes +ou-
(sqrt(2)), b=0, o mesmo ocorre com a simétrica.
como o termo independente (c) é o ponto onde a parábola corta o eixo x,
basta ver qual o simétrico de 2 em relação a reta y=-2 e pronto!

Qualquer dúvida, manda outra msg.

Espero ter ajudado,
Rossi

- Original Message -
From: "aryqueirozq" <[EMAIL PROTECTED]>
To: <[EMAIL PROTECTED]>
Sent: Tuesday, May 11, 2004 10:47 PM
Subject: [obm-l] Duvidas


>
>
>  Alguém poderia me ajudar nessas duas questoes.
>
>   Agradeço desde de já.
>
> 1)Os dados experimentais da tabela a seguir
> correspondem às concentrações de uma substância química
> medida em intervalos de 1 segundo. Assumindo que a
> linha que passa pelos três pontos experimentais é uma
> parábola, tem-se que a concentração (em moles) após 2,5
> segundos é:
>
> Tempo (s)Concentração (moles)
> 1 3,00
> 2 5,00
> 3 1,00
>
> a) 3,60
> b) 3,65
> c) 3,70
> d) 3,75
> e) 3,80
>
> 2)O gráfico da função quadrática y=ax2+bx+c, x real, é
> simétrico ao gráfico da parábola y=2-x2 com relação à
> reta de equação cartesiana y= -2. Determine o valor de
> 8a+b+c.
> a) - 4
> b) 1/2
> c) 2
> d) 1
> e) 4
>
>
>
>
>
>
>
>
> __
> Acabe com aquelas janelinhas que pulam na sua tela.
> AntiPop-up UOL - É grátis!
> http://antipopup.uol.com.br/
>
>
>
> =
> Instruções para entrar na lista, sair da lista e usar a lista em
> http://www.mat.puc-rio.br/~nicolau/olimp/obm-l.html
> =
>
>


=
Instruções para entrar na lista, sair da lista e usar a lista em
http://www.mat.puc-rio.br/~nicolau/olimp/obm-l.html
=


[obm-l] Re: [obm-l] correção da resolução doproblema(em tempo)

2004-05-11 Por tôpico Fellipe Rossi
aqui tb...
chegaram 4 de cada das ultimas 2 q vc mandou


- Original Message -
From: "Claudio Buffara" <[EMAIL PROTECTED]>
To: <[EMAIL PROTECTED]>
Sent: Tuesday, May 11, 2004 4:25 PM
Subject: Re: [obm-l] correção da resolução doproblema(em tempo)


> Oi, Vieira:
>
> O seu computador deve estar com algum problema pois eh a sexta vez que
> recebo esta mensagem.
>
> []s,
> Claudio.
>
> on 11.05.04 15:40, [EMAIL PROTECTED] at [EMAIL PROTECTED] wrote:
>
> > Em 11 May 2004, [EMAIL PROTECTED] escreveu:
> >
> >> Em 11 May 2004, [EMAIL PROTECTED] escreveu:
> >>
> >>> Vê se vcs podem me ajudar com esse probleminha:
> >>>
> >>> Se x^2 + y^2 = 9797, onde x e y são inteiros positivos
> >>> tais que x>y, existem exatamente dois pares ordenados
> >>> de inteiros (x,y) que satisfazem tal equação.A soma das
> >>> coordenadas deste dois pares é :
> >>> a)220
> >>> b)240
> >>> c)260
> >>> d)280
> >>> e)300
> >>> Sabemos que x >y,como x^2+y^2 é ímpar x e y tem paridades
> >> diferentes,sabemos também que x é estritamente < que 98 pois 98^2=9604
e
> >> y^2=193 mas y é inteiro positivo logo y é estritamente > que 14.Se x
for
> >> ímpar 9797-x^2 terá os finais 6,2 logo, testaremos x ímpar de final 1
ou 9
> >> 66 e < 98.Se x é par 9797-x^2 terá os finais 7,3,1 e os
> >> que estabelecem finais 1 são para x terminados em 4 ou 6.E ainda x deve
ser
> >>> que 66 pois como x>y y pode ser no máximo 65 e se x=66 assim
> >> x^2+y^2<9797.As únicas tentativas que você deve fazer para x são
> >> 69,71,74,76,79,81,84,86,89,94,96.E os únicos pares ordenados possíveis
são
> >> (86,49),(94,31)cuja soma nos dá 260.
> >> Ass:vieira
> >>>

>
>
> =
> Instruções para entrar na lista, sair da lista e usar a lista em
> http://www.mat.puc-rio.br/~nicolau/olimp/obm-l.html
> =
>
>


=
Instruções para entrar na lista, sair da lista e usar a lista em
http://www.mat.puc-rio.br/~nicolau/olimp/obm-l.html
=


Re: [obm-l] Demonstracoes no ensino medio

2004-05-09 Por tôpico Fellipe Rossi
a maioria, no meu modo de ver, exige cálculos.
Isso resulta em, quando pedem-se demonstrações, mesmo as mais simples, serem
questões consideradas difíceis. Até mesmo para questões literais os
vestibulando encontram dificuldades.

Aproveitando que fez referência sobre IME/ITA, vou propor uma questão que vi
em uma lista preparatoria e achei interessante:


**
Tome no plano xOy, A(a,0) e B(o,b)
Sejam A'(a+d,0) e B' (0,b+d), d Real.

Mostre que todas as mediatrizes dos segmentos A'B' Se interceptam em um
ponto.
**

Rossi


- Original Message -
From: "Claudio Buffara" <[EMAIL PROTECTED]>
To: "Lista OBM" <[EMAIL PROTECTED]>
Sent: Sunday, May 09, 2004 9:43 PM
Subject: [obm-l] Demonstracoes no ensino medio


> Esta eh uma pergunta para quem estah cursando ou lecionando matematica no
> ensino medio "normal" (ou seja, excluindo cursos de preparacao para
> olimpiadas ou vestibulares muito puxados como IME e ITA):
>
> Em provas e exercicios propostos exige-se que os alunos apresentem
> demonstracoes de resultados matematicos ou apenas pede-se que eles
calculem
> coisas?
>
> []s,
> Claudio.
>
> =
> Instruções para entrar na lista, sair da lista e usar a lista em
> http://www.mat.puc-rio.br/~nicolau/olimp/obm-l.html
> =
>
>


=
Instruções para entrar na lista, sair da lista e usar a lista em
http://www.mat.puc-rio.br/~nicolau/olimp/obm-l.html
=


Fw: [obm-l] 8a.cone sul

2004-05-09 Por tôpico Fellipe Rossi
Desculpem!! Faltou dizer que por congruência PT = TR


- Original Message -
From: "Fellipe Rossi" <[EMAIL PROTECTED]>
To: <[EMAIL PROTECTED]>
Sent: Sunday, May 09, 2004 9:50 PM
Subject: Re: [obm-l] 8a.cone sul


> Vamos la! :)
> Vejamos se ta certo
>
> Dificil explicar sem um desenho, aqueles q forem pacientes por favor lapis
e
> papel :)
>
> Assuma que o ponto onde a paralela ao diametro corta BR seja S
>
> Pegue o triângulo AOP, de ângulos P=90, O=b A=a,
> Devido à reta paralela, o triãngulo QPT tem os mesmos ângulos.
> Então fazendo a semelhança e chamando AO de 2r, usando o fato de QP ser
> metade de OP, vemos que QT = r
>
> Logo, QT é base média do triângulo AOH (metade da base e paralela a base),
> então sabemos que OT = OH.
>
> Agora tome os triângulos ABR e AOP que são semelhantes visto que ABR tem
um
> lado sobre o diametro então é retãngulo.
>
> OBSQ é um paralelogramo, temos que QS = 2r, então TS = r
>
> Assim, como OPT e TRH são semelhantes (do tipo LaL), temos que (ângulo
TRH)
> é reto, o que nos leva a concluir que B, R e H são colineares.
>
> Válido?
>
> Abraços do Rossi
>
>
> > >>- Original Message -
> > >>From:
> > >>To: ;
> > >>Sent: Sunday, May 09, 2004 4:11 PM
> > >>Subject: Re: [obm-l] 8a.cone sul
> > >>
> > >>> Em 8 May 2004, [EMAIL PROTECTED] escreveu:
> > >>>
> > >>> >Show de bola.Seja C uma circunferência de centro O, AB um diâmetro
> dela
> > >e
> > >>R
> > >>> >um ponto qualquer em C distinto de A e de B.Seja P a interseçâo da
> > >>> >perpendicular traçada por O a AR.Sobre a reta OP se marca o ponto
> Q,de
> > >>> >maneira que QP é a metade de PO e Q não pertence ao segmento OP.Por
Q
> > >>traçamos
> > >>> a
> > >>> >paralela a AB que corta a reta AR em T.Chamamos de H o ponto de
> > >>interseção
> > >>> >das retas AQ e OT.Provar que H,R e B são colineares.
> > >>> >
>
>


=
Instruções para entrar na lista, sair da lista e usar a lista em
http://www.mat.puc-rio.br/~nicolau/olimp/obm-l.html
=


Re: [obm-l] 8a.cone sul

2004-05-09 Por tôpico Fellipe Rossi
Vamos la! :)
Vejamos se ta certo

Dificil explicar sem um desenho, aqueles q forem pacientes por favor lapis e
papel :)

Assuma que o ponto onde a paralela ao diametro corta BR seja S

Pegue o triângulo AOP, de ângulos P=90, O=b A=a,
Devido à reta paralela, o triãngulo QPT tem os mesmos ângulos.
Então fazendo a semelhança e chamando AO de 2r, usando o fato de QP ser
metade de OP, vemos que QT = r

Logo, QT é base média do triângulo AOH (metade da base e paralela a base),
então sabemos que OT = OH.

Agora tome os triângulos ABR e AOP que são semelhantes visto que ABR tem um
lado sobre o diametro então é retãngulo.

OBSQ é um paralelogramo, temos que QS = 2r, então TS = r

Assim, como OPT e TRH são semelhantes (do tipo LaL), temos que (ângulo TRH)
é reto, o que nos leva a concluir que B, R e H são colineares.

Válido?

Abraços do Rossi


> >>- Original Message -
> >>From:
> >>To: ;
> >>Sent: Sunday, May 09, 2004 4:11 PM
> >>Subject: Re: [obm-l] 8a.cone sul
> >>
> >>> Em 8 May 2004, [EMAIL PROTECTED] escreveu:
> >>>
> >>> >Show de bola.Seja C uma circunferência de centro O, AB um diâmetro
dela
> >e
> >>R
> >>> >um ponto qualquer em C distinto de A e de B.Seja P a interseçâo da
> >>> >perpendicular traçada por O a AR.Sobre a reta OP se marca o ponto
Q,de
> >>> >maneira que QP é a metade de PO e Q não pertence ao segmento OP.Por Q
> >>traçamos
> >>> a
> >>> >paralela a AB que corta a reta AR em T.Chamamos de H o ponto de
> >>interseção
> >>> >das retas AQ e OT.Provar que H,R e B são colineares.
> >>> >



=
Instruções para entrar na lista, sair da lista e usar a lista em
http://www.mat.puc-rio.br/~nicolau/olimp/obm-l.html
=


[obm-l] Re: [obm-l] Re: [obm-l] Questão de 2o. grau

2004-05-09 Por tôpico Fellipe Rossi



Sim, mas essas demonstrações exigem uma experiência 
raramente encontrada em alunos de 2o. grau.
 
Outra questão do tipo seria mostrar que pra n>1, 
n pertence a N, que (n^2)! > (n!)^2
 
No caso eu faria:
 
(n^2)! = 1*2*3*...*n^2 = 
1*2*3...*n*(n+1)*(n+2)*...*n^2
(n!)^2= 1*1*2*2*3*3*4*4*...*n*n = 
1*2*3...*n*1*2*3...*n 
 
como (n+1)(n+2)...n^2 > 
1*2*3...*n    pois n+1>1, n+2>2,..., n^2>n 
provamos
 
Porém é mais um tipo de resolução que raramente 
entra na cabeça de vestibulandos.
 
Oq é mais normal fazer seria:
(2^2)! = 24
(2!)^2 = 4
 
(3^2)! = 9!
(3!)^2 = 36
 
para valores maiores de n maior será a 
diferença...
 
O que eu queria saber é se uma Banca de vestibular 
aceita esses tipos de resolução.
 
Grato pela atenção!
Rossi

  - Original Message - 
  From: 
  Marcio Afonso A. Cohen 
  To: [EMAIL PROTECTED] 
  Sent: Sunday, May 09, 2004 6:14 PM
  Subject: [obm-l] Re: [obm-l] Questão de 
  2o. grau
  
      Em parte. Tudo que voce diz eh 
  verdade, mas eu exigiria uma explicacao um pouquinho melhor de pq n! eh maior 
  que n^2. Mas a ideia eh otima e funciona. Eu acho q faria algo como: p/ 
  n>3, 1! + 2! + ... + n! >= n! + (n-1)!+1 > n(n-1) + (n-1) + 1 = 
  n^2.
      Uma outra opcao eh olhar mod 
  10.
  
- Original Message ----- 
    From: 
Fellipe Rossi 
To: [EMAIL PROTECTED] 
Sent: Sunday, May 09, 2004 5:42 
PM
Subject: [obm-l] Questão de 2o. 
grau

Como vocês demonstrariam, para 2o. grau, que 

 
para n>=1, n pertence a Z. apenas n=1 e n=3 
são raízes da equação:
 
1!+2!+3!+...+n! = n^2 
 
Vocês aceitariam uma resolução que mostrasse, 
com exemplos (4!=24, 4^2=16 ; 5!=120, 5^2=25, e assim por diante...) que 
para n>=4. n! é maior que n^2 e que como o lado esquerdo da igualdade eh 
n!+valor positivo, ela vai ser sempre maior que o lado direito para n>=4, 
e substituindo n por 1, 2 e 3 chegamos q apenas 1 e 3 são 
raizes?
 
Essa qustão caiu, se não me engano, na prova 
específica da UFRJ 1992.
 
Abraços! 
Rossi


[obm-l] Questão de 2o. grau

2004-05-09 Por tôpico Fellipe Rossi



Como vocês demonstrariam, para 2o. grau, que 

 
para n>=1, n pertence a Z. apenas n=1 e n=3 são 
raízes da equação:
 
1!+2!+3!+...+n! = n^2 
 
Vocês aceitariam uma resolução que mostrasse, com 
exemplos (4!=24, 4^2=16 ; 5!=120, 5^2=25, e assim por diante...) que para 
n>=4. n! é maior que n^2 e que como o lado esquerdo da igualdade eh n!+valor 
positivo, ela vai ser sempre maior que o lado direito para n>=4, e 
substituindo n por 1, 2 e 3 chegamos q apenas 1 e 3 são raizes?
 
Essa qustão caiu, se não me engano, na prova 
específica da UFRJ 1992.
 
Abraços! 
Rossi


Re: [obm-l] 8a.cone sul

2004-05-09 Por tôpico Fellipe Rossi
o treho q está confuso eh o da perpendicular a O.

no caso, P é a inteseeção da reta AR traçada por O (o "pé" da perpendicular)
ou é a interseção da reta perpendicular a AB por O?

eu creio q seja a primeira opção :)
- Original Message -
From: <[EMAIL PROTECTED]>
To: <[EMAIL PROTECTED]>; <[EMAIL PROTECTED]>
Sent: Sunday, May 09, 2004 4:11 PM
Subject: Re: [obm-l] 8a.cone sul


> Em 8 May 2004, [EMAIL PROTECTED] escreveu:
>
> >Show de bola.Seja C uma circunferência de centro O, AB um diâmetro dela e
R
> >um ponto qualquer em C distinto de A e de B.Seja P a interseçâo da
> >perpendicular traçada por O a AR.Sobre a reta OP se marca o ponto Q,de
> >maneira QP é a metade de PO e Q não pertence ao segmento OP.Por Q
traçamos
> a
> >paralela a AB que corta a reta AR em T.Chamamos de H o ponto de
interseção
> >das retas AQ e OT.Provar que H,R e B são colineares.
> >
> >_
> >Voce quer um iGMail protegido contra vírus e spams?
> >Clique aqui: http://www.igmailseguro.ig.com.br
> >Ofertas imperdíveis! Link: http://www.americanas.com.br/ig/
> >
> >=
> >Instruções para entrar na lista, sair da lista e usar a lista em
> >http://www.mat.puc-rio.br/~nicolau/olimp/obm-l.html
> >=
> >
> >--
>
> _
> Voce quer um iGMail protegido contra vírus e spams?
> Clique aqui: http://www.igmailseguro.ig.com.br
> Ofertas imperdíveis! Link: http://www.americanas.com.br/ig/
>
> =
> Instruções para entrar na lista, sair da lista e usar a lista em
> http://www.mat.puc-rio.br/~nicolau/olimp/obm-l.html
> =
>
>


=
Instruções para entrar na lista, sair da lista e usar a lista em
http://www.mat.puc-rio.br/~nicolau/olimp/obm-l.html
=


Re: [obm-l] 8a.cone sul

2004-05-09 Por tôpico Fellipe Rossi
ta mto confuso o enunciado
tem certeza q transcreveu corretamente?


- Original Message -
From: <[EMAIL PROTECTED]>
To: <[EMAIL PROTECTED]>; <[EMAIL PROTECTED]>
Sent: Sunday, May 09, 2004 4:11 PM
Subject: Re: [obm-l] 8a.cone sul


> Em 8 May 2004, [EMAIL PROTECTED] escreveu:
>
> >Show de bola.Seja C uma circunferência de centro O, AB um diâmetro dela e
R
> >um ponto qualquer em C distinto de A e de B.Seja P a interseçâo da
> >perpendicular traçada por O a AR.Sobre a reta OP se marca o ponto Q,de
> >maneira QP é a metade de PO e Q não pertence ao segmento OP.Por Q
traçamos
> a
> >paralela a AB que corta a reta AR em T.Chamamos de H o ponto de
interseção
> >das retas AQ e OT.Provar que H,R e B são colineares.
> >
> >_
> >Voce quer um iGMail protegido contra vírus e spams?
> >Clique aqui: http://www.igmailseguro.ig.com.br
> >Ofertas imperdíveis! Link: http://www.americanas.com.br/ig/
> >
> >=
> >Instruções para entrar na lista, sair da lista e usar a lista em
> >http://www.mat.puc-rio.br/~nicolau/olimp/obm-l.html
> >=
> >
> >--
>
> _
> Voce quer um iGMail protegido contra vírus e spams?
> Clique aqui: http://www.igmailseguro.ig.com.br
> Ofertas imperdíveis! Link: http://www.americanas.com.br/ig/
>
> =
> Instruções para entrar na lista, sair da lista e usar a lista em
> http://www.mat.puc-rio.br/~nicolau/olimp/obm-l.html
> =
>
>


=
Instruções para entrar na lista, sair da lista e usar a lista em
http://www.mat.puc-rio.br/~nicolau/olimp/obm-l.html
=


[obm-l] Re: [obm-l] dúvida

2004-05-09 Por tôpico Fellipe Rossi



cada corte aparecem 2 novos pedaços
logo pra termos catorze precisamos de 7 
cortes
como ele ja fez 1, serão necessários mais 
6
o ângulo do setor será 360 graus / 14 = 
aproximadamente 26 graus

  - Original Message - 
  From: 
  TSD 
  To: [EMAIL PROTECTED] 
  Sent: Saturday, May 08, 2004 6:35 
PM
  Subject: [obm-l] dúvida
  
  
  17 - Uma pizza de 
  formato circular será cortada em fatias pormeio de cortes quesão segmentos de 
  reta que passam pelo"centro" da pizza e medem um diâmetro. A figura a seguir 
  mostra que, com um corte, a pizza fica dividida em duas fatias iguais.( é uma 
  circunferencia com um traço diametro)
  Para se obter quatorze fatias iguais, a partir do 
  primeiro corte,serão necessários cortes adicionais na pizza. Assinale o item 
  que indica o número de cortes adicionais que terão de ser feitos e o ângulo 
  aproximado que cada fatia, imaginada como um setor circular, terá:
  (A) 8 e 30o;(B) 7 e 26o ;(C) 6 e 25o;(D) 7 e 30o; (E) 8 e 26o.
   
   


[obm-l] Re: [obm-l] Re:[obm-l] Essa é fácil ???? sim ou não ????

2004-05-07 Por tôpico Fellipe Rossi
Acho que quando vc fez "Par+Par" etc etc, vc deixou de considerar, por
exemplo, que podem ser 1,4 + 1,6
- Original Message -
From: "rickufrj" <[EMAIL PROTECTED]>
To: "obm-l" <[EMAIL PROTECTED]>
Sent: Friday, May 07, 2004 1:04 PM
Subject: [obm-l] Re:[obm-l] Essa é fácil  sim ou não 


> 2 ^ x+ 1, sabendo que f(a)= 4f(b) . Para quais reais
> valores de a e b ?
>
>
> =
>
> Se f(x)=2^x + 1
> E queremos a e b , tal que :
> f(a)=4f(b) , entao:
> 2^a + 1 = 2^(2+b) + 4
> 2^a - 2^(2+b) = 3 (i)
> Temos uma expressao do tipo :
> 2^k - 2^t = 3
> Sabendo que ;
> par +/- impar = impar ,
> par +/- par = par . Podemos dizer ainda que 2^k ou 2^t
> so e impar quando k ou t for zero .
> Entao dividimos o problema em duas partes :
> 1°)k=0
> Concluimos que nao existe t , consequentemente esse
> caso sai fora da analise .
> 2°)t=0
> Nesse caso encontramos
> 2^k = 4 , k = 2
>
> Voltando a (i):
> 2+b=0 e a=2 , b=-2 e a=2.
>
>
> Abraco
> Luiz H. Barbosa
>
> __
> Acabe com aquelas janelinhas que pulam na sua tela.
> AntiPop-up UOL - É grátis!
> http://antipopup.uol.com.br/
>
>
>
> =
> Instruções para entrar na lista, sair da lista e usar a lista em
> http://www.mat.puc-rio.br/~nicolau/olimp/obm-l.html
> =
>
>


=
Instruções para entrar na lista, sair da lista e usar a lista em
http://www.mat.puc-rio.br/~nicolau/olimp/obm-l.html
=


Re: [obm-l] Curiosidade OFF-TOPIC

2004-05-07 Por tôpico Fellipe Rossi



Sempre me perguntam isso no trabalho 
hehehe
 
Quanto a hora podemos pensar que um relógio de 
ponteiros tem 360 graus = 60 minutos de hora (ponteiro grande) e 30 graus = 
1 hora (ponteiro menor)... Então 1 grau não corresponde a 60 minutos de 
hora.
 
Eu não tenho certeza mas oq eu respondo é que 
Minutos significam 1/60 de qualquer coisa. Como se fosse Décimo, ou 
Centésimo
 
Tipo uma banana pode ser dividida em 60 minutos de 
banana ehehe
 
Mas não tenho certeza se é isso.
 
 

  - Original Message - 
  From: 
  Alan Pellejero 
  To: [EMAIL PROTECTED] 
  Sent: Wednesday, May 05, 2004 11:49 
  AM
  Subject: [obm-l] Curiosidade 
  OFF-TOPIC
  
  Sobre os ângulos:
   
  1' = 60 "
  60' = 1º
   
  Essas são as relações, porém, há alguma relação entre o grau ser 60 
  minutos e uma hora também o ser?
   
  Muito obrigado,
  Alan Pellejero
  
  
  Yahoo! 
  Messenger - Fale com seus amigos online. Instale 
  agora!


Re: [obm-l] corrida...

2004-05-04 Por tôpico Fellipe Rossi
Por lógica, Letra A.

As possibilidades - com Rubino logo atrás de David- sao
I) Ralf  David  Rubinho
II) David Rubinho Ralf

Porém,
B) está errada pois se Ralf ganha, temos a situação II, o que daria a letra
C como certa também.
C) anula-se com B
D) Ralf em segundo eh impossivel pois não poderiamos ter Rubinho logo atrás
de David.

Logo temos a situação I.

Abraços!
Rossi

- Original Message -
From: "niski" <[EMAIL PROTECTED]>
To: <[EMAIL PROTECTED]>
Sent: Tuesday, May 04, 2004 1:51 PM
Subject: [obm-l] corrida...


> Pessoal, levei um tempinho para resolver esta questão, e cheguei a
> conclusao que a resposta é a)
> Gostaria de saber a opinião dos colegas da lista. Se alguem tem uma boa
> solucao e etc
>
> O seguinte trecho de artigo de um jornal local relata uma corrida
> beneficente de bicicletas: "Alguns segundos após a largada, Ralf tomou a
> liderança, seguido de perto por David e Rubinho, nesta ordem. Daí
> em diante, eles não mais deixaram as primeiras três posições e, em
> nenhum momento da corrida, estiveram lado a lado mais do que dois
> competidores. A liderança, no entanto, mudou de mãos nove vezes entre os
> três, enquanto que em mais oito ocasiões diferentes aqueles que corriam
> na segunda e terceira posições trocaram de lugar entre si. Após o
> término da corrida, Rubinho reclamou para nossos repórteres que
> David havia conduzido sua bicicleta de forma imprudente pouco antes da
> bandeirada de chegada. Desse modo, logo atrás de David, Rubinho não pôde
> ultrapassá-lo no final da corrida."
>
> Com base no trecho acima, você conclui que
> A) David ganhou a corrida.
> B) Ralf ganhou a corrida.
> C) Rubinho chegou em terceiro lugar.
> D) Ralf chegou em segundo lugar.
> E) não é possível determinar a ordem de chegada, porque o trecho não
> apresenta uma descrição matematicamente
> correta.
>
> Obrigado
>
> --
> Niski - http://www.linux.ime.usp.br/~niski
>
> [upon losing the use of his right eye]
> "Now I will have less distraction"
> Leonhard Euler
>
> =
> Instruções para entrar na lista, sair da lista e usar a lista em
> http://www.mat.puc-rio.br/~nicolau/olimp/obm-l.html
> =
>
>


=
Instruções para entrar na lista, sair da lista e usar a lista em
http://www.mat.puc-rio.br/~nicolau/olimp/obm-l.html
=


[obm-l] Re: [obm-l] Progressão! Algo errado!

2004-05-03 Por tôpico Fellipe Rossi



Carlos, um esquema das roseiras:
 
Fonte15m__o_o_o_o_o_o__o
 
Primeira "ida-e-volta" ele anda 15+2+15+2 
= 34
Segunda ele anda 15+2+3+15+2+3 = 40
Terceira ele anda 15+8+15+8 = 46
e assim vai, formando uma PA de 
razão=6 e 20 termos (60/3)
a_20 = 34 + 19*6 = 148
Ele vai andar o somatório das distâncias = (34 
+148) * 20/2 = 1820 metros.
 
Espero ter ajudado,
Rossi

  - Original Message - 
  From: 
  Carlos 
  Alberto 
  To: [EMAIL PROTECTED] 
  Sent: Monday, May 03, 2004 8:08 AM
  Subject: [obm-l] Progressão! Algo 
  errado!
  
  Um jardineiro tem que regar 60 roseiras plantadas ao londo de uma vereda 
  retílinea e distante 1m uma da outra. Ele enche seu regador numa fonte situada 
  na mesma vereda, a 15m da primeira roseira, e a cada viagem rega 3 roseiras. 
  Começando e terminando na fonte, qual é o percurso total que ele terá que 
  caminhar ate regar todas as roseiras?
   
  Bom, o resultado da minha solução não está batendo com a do livro.
  Provavelmente estou cometendo um equívoco, em algum lugar.
   
  Desejaria que algum me apontasse o erro. Pois não consigo enchergar onde 
  estou errando.
   
  Minha Resolução:
   
  Imaginei da seguinte forma, o jardineiro tem que encher o regador 
  percorrer uma distancia regar 3 roseiras, voltar percorrendo a mesma distancia 
  para encher o regador, depois percorrer a distancia anterior mais uma 
  distancia y para regar mais 3 roseiras.
   
  Logo as distancias forma uma P.A.
   
  P.A. de razão = 6 e a_1 = 34
  a_60 = 388
   
  A PA seria: (34,40,46,52,...,388)
   
  Logo o percurso seria a soma das distancias percorridas.
   
  S_60 = (60(34+388))/2 = 12 660 metros.
   
  O livro me dá o Resultado de 1 820 metros.
   
  Totalmente distinto da minha resolução.
  Algum erro grave eu provavelmente estou cometendo.
   
  Queria que alguem me ajudasse...
  Desde já agradeço,
  Carlos
  
  
  Yahoo! 
  Messenger - Fale com seus amigos online. Instale 
  agora!


Re: Alerta de spam:Re: [obm-l] Notification

2004-04-29 Por tôpico Fellipe Rossi



Voce mandou por 2 vezes ja um arquivo chamado 
JOKE.com q o meu norton pegou como virus!
Atençao pessoal da LISTA!!! Não abram o arquivo 
JOKE.COM!!
 
Rossi

  - Original Message - 
  From: 
  [EMAIL PROTECTED] 
  
  To: [EMAIL PROTECTED] 
  Sent: Thursday, April 29, 2004 6:17 
  PM
  Subject: Alerta de spam:Re: [obm-l] 
  Notification
  Por que ? 
  Meu computador nao esta com problemas ? Achei estranho eu ter recebido um 
  e-mail de notificacao sendo que o remetente era meu mesmo ?! Em uma 
  mensagem de 29/4/2004 10:26:00 Hora padrão leste da Am. Sul, [EMAIL PROTECTED] 
  escreveu: 
  Fael, acho que vc tá com virus... 
- Original Message - From: Faelccmm To: Obm-l Sent: Thursday, 
  April 29, 2004 9:18 AM Subject: [obm-l] Notification 
  


Re: [obm-l] Notification

2004-04-29 Por tôpico Fellipe Rossi



Fael, acho que vc tá com virus...

  - Original Message - 
  From: 
  Faelccmm 
  To: Obm-l 
  Sent: Thursday, April 29, 2004 9:18 
  AM
  Subject: [obm-l] Notification
  


Re: [obm-l] Geometria Plana - Desafio (?)

2004-04-29 Por tôpico Fellipe Rossi
Boromir não consigo entender nada da mensagem
Talvez voce esteja usando mtos caracteres "especiais"...

Rossi
- Original Message -
From: "Boromir" <[EMAIL PROTECTED]>
To: <[EMAIL PROTECTED]>
Sent: Thursday, April 29, 2004 12:14 AM
Subject: Re: [obm-l] Geometria Plana - Desafio (?)


> Vamos considerar a < b. Seja ainda P o ponto de encontro dos
> prolongamentos dos lados não paralelos DA e CB. Conforme o enunciado,
> [ABNM]=[NMDB] = S. ([figura] = área do figura) Vamos considerar [APB]=K.
> APB ~ MPN  (razão a/x, onde MN = x). A razão entre as áreas é o
quadrado
> da razão de semelhança, portanto (K+S)/K = (x/a)². Ainda temos que
> APB~DPC (razão a/b), portanto (K+2S)/K = (b/a)².
> Escrevendo melhor as equações acima, temos:
> 1 + S/K = x²/a² -> S/K = (x²-a²)/a²
> 1 +2S/K = b²/a² -> 2S/K = (b²-a²)/a²
> Dividindo a segunda pela primeira equação temos:
> 2(x²-a²) = b²-a²
> 2x²=b²+a²
> x = SQRT{(a²+b²)/2}
>
> Se eu não errei as contas acho que é isso.
> []'s MP
>
> Em Ter, 2004-04-27 Ã s 18:42, Victor Machado escreveu:
> > Bom, esta questão foi um desafio para mim, não sei para os senhores :
> >
> > Dado um trapézio ABCD de bases AB= a e CD=b e os pontos M e N
> > pertencentes aos lados NÃfO-paralelos. Se o segmento MN divide esse
> > trapézio em dois outros trapézios equivalentes, calcule MN em função
> > dos lados AB=a e CD=b.
> >
> > Victor.
>
> =
> Instruções para entrar na lista, sair da lista e usar a lista em
> http://www.mat.puc-rio.br/~nicolau/olimp/obm-l.html
> =
>
>


=
Instruções para entrar na lista, sair da lista e usar a lista em
http://www.mat.puc-rio.br/~nicolau/olimp/obm-l.html
=


Re: [obm-l] Questao da Eureka 01

2004-04-29 Por tôpico Fellipe Rossi
ï


Certamente!! =D

  - Original Message - 
  From: 
  [EMAIL PROTECTED] 
  
  To: [EMAIL PROTECTED] 
  Sent: Wednesday, April 28, 2004 12:13 
  AM
  Subject: Re: [obm-l] Questao da Eureka 
  01
  
  Se a diferenÃa entre dois primos à 3, entÃo um à 
  par, outro Ãmpar.
   
  Aà à automÃtico que um deles à 2...
   
  234
  
- Original Message - 
From: 
Fellipe Rossi 
To: [EMAIL PROTECTED] 
Sent: Tuesday, April 27, 2004 11:52 
PM
Subject: Re: [obm-l] Questao da Eureka 
01

bom o produto das raÃzes à -3/a e a soma 
-b/a
uma delas à -1
logo -1xp=-3/a -> p=3/a.
p-1 = -b/a
3-a = -b
a-b = 3
 
Primos: {2,3,5,7,11,13,17,19,...}
 
Como a e b sao primos com diferenÃa=3, a=5 e 
b=2  (nao precisamos pensar em numeros grandes visto que o maior valor 
do enunciado eh 85 e 11^2 ja seria maior do que isto, porÃm nÃo sei como 
provar que os Ãnicos primos em q a diferenÃa vale 3 serÃo 2 e 
5)
 
logo, a^2 + b^2 = 29.
 
Acho que à isso.
 
alguÃm poderia dar uma olhada no parÃnteses? 
:)
 
AbraÃos,
Rossi

  - Original Message - 
  From: 
  [EMAIL PROTECTED] 
  To: [EMAIL PROTECTED] 
  Sent: Tuesday, April 27, 2004 7:04 
  PM
  Subject: [obm-l] Questao da Eureka 
  01
  Ola pessoal, Poderiam me explicar como se resolve 
  esta: 1) A equacao do 2Â grau ax^2 +  bx â 3 = 0 tem â1 como 
  uma de suasraizes. Sabendo que os coeficientes a e b sao numeros 
  primos positivos, podemos afirmar que a^2 + b^2 eh igual a:a) 29 
  b) 89 c) 17 d) 13 e) 53Ps: Alguem poderia me enviar a figura a que 
  se refere a questao de treinamento (numero 02) da eureka 01 ? Fiz o 
  download da revista, mas nao aparece esta figura. 
  


Re: [obm-l] A BICICLETA AZARENTA!

2004-04-29 Por tôpico Fellipe Rossi
Acho que ninguém respondeu essa ainda.

Vamos considerar que o Registro da bicicleta tenha N digitos
então, de 00...0 até 99...9, temos 10^n números.

Para calcular aqueles em que figuram algarismos 8, vamos fazer 1-P(não 8)

de 00...0 até 99...9, sem figurar o 8, temos 9^n

Então P8 = 1-(9^n/10^n).
Sendo assim, para
n=1, P8=10%
n=2, P8=19%
n=3, P8=27,1%
.
.
.
Conforme aumentamos o n, P8 aumenta.
***Ou seja, só será 10% para n=1, logo a afirmação do rapaz está
incorreta.***


Agora queremos que P8>50%, ou seja, 1-(9/10)^n > 1/2
Logo, (9/10)^n < 1/2, então
para n=6, (9/1)^6 = 0,5314...
e n=7, (9/10)^7 = 0,4782...

***A partir de 7 algarismos no Registro, P8>50%***

Certo?

Abraços do Rossi

- Original Message -
From: <[EMAIL PROTECTED]>
To: <[EMAIL PROTECTED]>
Sent: Friday, April 23, 2004 8:10 PM
Subject: [obm-l] A BICICLETA AZARENTA!


> OK! Qwert e demais colegas! Vou revisar o tal enunciado que tomei nota a
pelo
> menos duas décadas atrás. Pelo sim, pelo não, muito grato pela sua
correção!
>
> Artur é um jovem muito superticioso. Certa feita, quando Artur ganhou uma
> bicicleta, foi advertido: cuidado com o OITO. Na realidade, tratava-se de
uma
> recomendação com respeito a um tipo de avaria própria de algumas
bicicletas.
> Mas, Artur interpretou que poderia estar condenado a algum contratempo se,
no
> número (da série) de fabricação da bicicleta, figurasse algum algarismo
OITO.
> Após refletir um pouco, Artur tranquilizou-se com o seguinte raciocínio:
> qualquer que seja o número de Registro, ele tem que ser formado com
algarismos
> que vão de 0 a 9; destes, somente o OITO é azíago; logo, em cada 10 casos,
> existe apenas um em que a matrícula é "azarenta". Portanto, a chance de
> conseguir uma bicicleta "sortuda" é de 90%. Está correta a conclusão de
Artur?
> Se não, a partir de quantos algarismos no Registro teremos mais
> números "azarentos" do que números "de sorte"?
>
>
> Um abraço à todos e bom final de semana!
>
>
>
> 
> WebMail UNIFOR - http://www.unifor.br
> =
> Instruções para entrar na lista, sair da lista e usar a lista em
> http://www.mat.puc-rio.br/~nicolau/olimp/obm-l.html
> =
>
>


=
Instruções para entrar na lista, sair da lista e usar a lista em
http://www.mat.puc-rio.br/~nicolau/olimp/obm-l.html
=


Re: [obm-l] Questao da Eureka 01 - Desculpem!!

2004-04-27 Por tôpico Fellipe Rossi
Title: Re: [obm-l] Questao da Eureka 01



Ops!!
Eram tantas msgns que acabei deixando essa passar e 
mandei denovo a mesma coisa
 
Sorry =/
 
Rossi

  - Original Message - 
  From: 
  Claudio Buffara 
  To: [EMAIL PROTECTED] 
  Sent: Tuesday, April 27, 2004 9:44 
  PM
  Subject: Re: [obm-l] Questao da Eureka 
  01
  on 27.04.04 19:04, [EMAIL PROTECTED] at [EMAIL PROTECTED] wrote:
  Ola pessoal, Poderiam me explicar como se 
resolve esta: 1) A equacao do 2º grau ax^2 +  bx ­ 3 = 0 
tem ­1 como uma de suasraizes. Sabendo que os coeficientes a e b sao 
numeros primos positivos, podemos afirmar que a^2 + b^2 eh igual 
a:a) 29 b) 89 c) 17 d) 13 e) 53as raizes sao -1 a u ==> 
u - 1 = -b/a  e  -u = -3/a ==>3/a - 1 = -b/a 
==>3 - a = -b ==>a = b + 3 ==>a > b  e  a, 
b tem paridades distintas.Como eles sao primos positivos, soh pode 
ser b = 2 e a = 5 ==>a^2 + b^2 = 25 + 4 = 
29[]s,Claudio.


Re: [obm-l] Questao da Eureka 01

2004-04-27 Por tôpico Fellipe Rossi
ï


bom o produto das raÃzes à -3/a e a soma 
-b/a
uma delas à -1
logo -1xp=-3/a -> p=3/a.
p-1 = -b/a
3-a = -b
a-b = 3
 
Primos: {2,3,5,7,11,13,17,19,...}
 
Como a e b sao primos com diferenÃa=3, a=5 e 
b=2  (nao precisamos pensar em numeros grandes visto que o maior valor do 
enunciado eh 85 e 11^2 ja seria maior do que isto, porÃm nÃo sei como provar que 
os Ãnicos primos em q a diferenÃa vale 3 serÃo 2 e 5)
 
logo, a^2 + b^2 = 29.
 
Acho que à isso.
 
alguÃm poderia dar uma olhada no parÃnteses? 
:)
 
AbraÃos,
Rossi

  - Original Message - 
  From: 
  [EMAIL PROTECTED] 
  
  To: [EMAIL PROTECTED] 
  Sent: Tuesday, April 27, 2004 7:04 
  PM
  Subject: [obm-l] Questao da Eureka 
  01
  Ola pessoal, Poderiam me explicar como se resolve esta: 
  1) A equacao do 2Â grau ax^2 +  bx â 3 = 0 tem â1 como uma de 
  suasraizes. Sabendo que os coeficientes a e b sao numeros primos 
  positivos, podemos afirmar que a^2 + b^2 eh igual a:a) 29 b) 89 c) 17 
  d) 13 e) 53Ps: Alguem poderia me enviar a figura a que se refere a 
  questao de treinamento (numero 02) da eureka 01 ? Fiz o download da revista, 
  mas nao aparece esta figura. 


Re: Re:[obm-l] Binomios... Duvida ( interessante )

2004-04-27 Por tôpico Fellipe Rossi
Só pra esclarescer um pouco mais
O que o rick quis dizer eh q qdo multiplicamos o x^0 com o x^3 obtemos um
coeficiente de grau 3, o mesmo para 1 e 2 e pra todos q ele exemplificou.
Assim, seu trabalho sera achar cada coeficiente em cada polinomio e depois
multiplicar os q equivalerão a 3...
meio braçal isto hehe


- Original Message -
From: "rickufrj" <[EMAIL PROTECTED]>
To: "obm-l" <[EMAIL PROTECTED]>
Sent: Tuesday, April 27, 2004 11:17 PM
Subject: Re:[obm-l] Binomios... Duvida ( interessante )


> -- Início da mensagem original ---
>
>   De: [EMAIL PROTECTED]
> Para: [EMAIL PROTECTED]
>   Cc:
> Data: Tue, 27 Apr 2004 22:44:45 -0300
>  Assunto: [obm-l] Binomios... Duvida ( interessante )
>
> > 1) Determinar o coeficiente de x^3 no
> desenvolvimento de ( 2x - 3 )^4 . ( x + 2 )^5
> >
> >
> > Alguem pode me explicar o caminho ?
> > Abração!
>
> 
> Olhe para :
> x^3=(x^0)*(x^3)=(x^1)*(x^2)=(x^2)*(x^1)=(x^3)*(x^0)
> Com isso vc calcula os coeficientes de cada um e soma !
> Abraços
> Luiz H. Barbosa
>
> __
> Acabe com aquelas janelinhas que pulam na sua tela.
> AntiPop-up UOL - É grátis!
> http://antipopup.uol.com.br/
>
>
>
> =
> Instruções para entrar na lista, sair da lista e usar a lista em
> http://www.mat.puc-rio.br/~nicolau/olimp/obm-l.html
> =
>
>


=
Instruções para entrar na lista, sair da lista e usar a lista em
http://www.mat.puc-rio.br/~nicolau/olimp/obm-l.html
=


[obm-l] Re: [obm-l] Re:[obm-l] Re: [obm-l] A menor bissetriz e o maior lado de um triângulo

2004-04-25 Por tôpico Fellipe Rossi
Certamente,
Não atribui valores, apenas considerei a>b>c pra efeito de demonstração.
Assim como eu poderia ter dito q b>c>a e assim por diante
Apenas tomei isto como ponto de partida.
por exemplo, se eu tivesse considerado q b>c>a o lado maior seria outro, e
eu teria que provar que outra bissetriz seria a menor.

Porém tem razão não provei que 2a+c>a+2b.
Então está furada minha demonstração :(

Vou continuar tentando

- Original Message -
From: "rafsanco" <[EMAIL PROTECTED]>
To: "obm-l" <[EMAIL PROTECTED]>
Sent: Sunday, April 25, 2004 5:04 PM
Subject: [obm-l] Re:[obm-l] Re: [obm-l] A menor bissetriz e o maior lado de
um triângulo


> Olá Fellipe !
>
> Compreendi sua demonstração, porém tenho comentários a
> fazer: no início você supôs que 2a > 2b > 2c e daí
> concluiu que BC seria o maior lado do triângulo. É
> lícito considerar 2a > 2b > 2c ? Penso que certo seria
> dizer primeiramente que BC é o maior lado, logo 2a > 2b
> e 2a > 2c (e assim nada poderíamos inferir a respeito
> de 2b e 2c, ou seja, quem é maior ou menor), não ?
> Outra coisa: 2a + c > a + 2b ? Por quê ? Não entendi
> isso muito bem. De qualquer forma agradeço sua ajuda.
>
> Abraços,
>
> Rafael.
>
> __
> Acabe com aquelas janelinhas que pulam na sua tela.
> AntiPop-up UOL - É grátis!
> http://antipopup.uol.com.br/
>
>
>
> =
> Instruções para entrar na lista, sair da lista e usar a lista em
> http://www.mat.puc-rio.br/~nicolau/olimp/obm-l.html
> =
>
>


=
Instruções para entrar na lista, sair da lista e usar a lista em
http://www.mat.puc-rio.br/~nicolau/olimp/obm-l.html
=


[obm-l] Re: [obm-l] Re:_[obm-l]__A_menor_bissetriz_e_o_maior_lado_de_um_triângulo

2004-04-25 Por tôpico Fellipe Rossi



Eu quis dizer que tenho um desenho, explicar a 
demonstração seria mais simples, pois não precisaria descrever cada passo e 
ficaria menor..
não estava me referindo a provar com desenho 
geometrico :)

  - Original Message - 
  From: 
  Johann Peter Gustav Lejeune 
  Dirichlet 
  To: [EMAIL PROTECTED] 
  Sent: Sunday, April 25, 2004 4:55 
PM
  Subject: Re: [obm-l] 
  Re:_[obm-l]__A_menor_bissetriz_e_o_maior_lado_de_um_triângulo
  
  Lembre-se de que geometria nao depende de desenho!Fellipe 
  Rossi <[EMAIL PROTECTED]> 
  wrote: 
  



Bom Rafael, eu tentei por esse 
caminho.
Esteja com lapis e papel para anotar direitinho 
hehehe eh meio grande.
 
Seja ABC o triangulo, o angulo interno de A=2a, 
B=2b e C=2c, onde 2a>2b>2c, logo o maior lado é o lado 
BC.
 
Agora tome I como incentro de ABC, M o pé da 
bissetriz relativa a BC, N o pé da bissetriz relativa a AC e P o pé da 
bissetriz relativa a AB.
 
Olhando para o triângulo AIB, como a>b 
podemos afirmar que BI>AI
Olhando para o triângulo AIC, como a>c 
podemos afirmar que CI>AI
 
Agora, se conseguirmos provar que IM
 
traçando os raios do círculo inscrito, formamos 
3 triangulos retângulos com I e os pés das bissetrizes.
analisando esses triangulos, podemos dizer, 
pela propriedade dos angulos extermos, que o angulo M vale a+2c e o ângulo N 
vale b+2c.
como 
a+2c > b+2c, 
então MI
Logo, ja sabemos que AM
 
tome o ângulo BPC, ele vale 2a+c.
tome o angulo AMC que vale a+2b  (ambos 
obtidos por ângulo externo)
Como BPC>AMC, o ângulo P do triângulo 
retângulo é menor do que o ângulo M, do outro triângulo 
retângulo.
Logo IP>MI
 
então, CP>AM
 
Creio que está provado que AM (bissetriz 
relativa ao maior lado) é menos do que BN e CP.
 
O que vocês acham (aqueles que tiveram 
paciencia de ler ate o fim eheh)
 
PS: com um desenho seria bem mais simples 
explicar.
 
Abraços do Rossi
 
 

  - Original Message - 
  From: 
  Johann Peter Gustav Lejeune 
  Dirichlet 
  To: [EMAIL PROTECTED] 
  Sent: Sunday, April 25, 2004 11:41 
  AM
  Subject: Re: [obm-l] A menor 
  bissetriz e o maior lado de um triângulo
  
  Acho que da para ir de trigonometria nao?Depois eu dou uma 
  olhada...rafsanco <[EMAIL PROTECTED]> 
  wrote: 
  Olá 
para todos !Deparei-me com um teorema de geometria euclidiana 
plana que dizia o seguinte: ao maior lado de um triângulo 
corresponde a menor bissetriz. Tentei prová-lo da seguinte forma 
(infelizmente não disponho de recursos visuais, então usem a 
imaginação ou esboçem o desenho num papel para compreenderem melhor 
o que digo): Seja ABC um triângulo qualquer, BC seu maior lado, I 
seu incentro, x a medida do angulo interno de vértice A, y a 
medida do ângulo interno de vértice B, z a medida do ângulo interno 
de vértice C, AM a bissetriz de x, BO a bissetriz de y e CN a 
bissetriz de z. Sabe-se que x > y e x > z uma vez que x é 
oposto a BC (suposto maior lado). Analisando o triângulo AIC, vê-se 
que x/2 > z/2, logo CI > AI. Observando o triângulo AIB é 
verdadeiro afirmar que x/2 > y/2, portanto BI >! ! AI. Ora IM, 
IN e IO são segmentos de reta congruentes, visto que são raios 
da circunferência inscrita a ABC, então BI + IO > AI + IM o que 
implica que BO > AM (BI + IO = BO e AI + IM = AM), assim como CI 
+ IN > AI + IM o que implica que CN > AM (CI + IN = CN e AI + 
IM = AM). Enfim, está demonstrada a tese AM < BO e AM < CN. A 
minha demonstração é válida ou há algo nela que a compromete 
(sei lá, algum argumento duvidoso, por exemplo) ? Vocês conhecem 
alguma outra maneira de se provar esse teorema ? Se sim, exponha-a 
por 
favor.Abraços,Rafael.__Acabe 
com aquelas janelinhas que pulam na sua tela.AntiPop-up UOL - É 
grátis!http://antipopup.uol.com.br/=Instruções 
para entrar na lista, sair da lista e usar a lista 
emhttp://www.mat.puc-rio.br/~nicolau/olimp/obm-l.html=
  
  
  TRANSIRE SVVM PECTVS MVNDOQVE 
  POTIRI
  CONGREGATI EX TOTO ORBE MATHEMATICI 
  OB SCRIPTA INSIGNIA TRIBVERE
  Fields Medal(John Charles 
  Fields)
  
  
  Yahoo! 
  Messenger - Fale com seus amigos online. Instale 
  agora!
  
  
  TRANSIRE SVVM PECTVS MVNDOQVE 
  POTIRI
  CONGREGATI EX TOTO ORBE MATHEMATICI OB 
  

Re: Re:[obm-l] CN 98

2004-04-25 Por tôpico Fellipe Rossi
Eu resolvi a questão sem e referido teorema.
Usei a mesma relação de existencia dos triangulos para as 2 equações.

[]'s
- Original Message -
From: "Victor Machado" <[EMAIL PROTECTED]>
To: <[EMAIL PROTECTED]>
Sent: Sunday, April 25, 2004 3:57 PM
Subject: Re: Re:[obm-l] CN 98


> Você poderia explicar este "Teorema da Envoltória" melhor ?
> Nunca ouvi falar.
> Victor.
> - Original Message -
> From: rickufrj <[EMAIL PROTECTED]>
> To: obm-l <[EMAIL PROTECTED]>
> Sent: Sunday, April 25, 2004 12:26 PM
> Subject: Re:[obm-l] CN 98
>
>
> > -- Início da mensagem original ---
> >
> >   De: [EMAIL PROTECTED]
> > Para: [EMAIL PROTECTED]
> >   Cc:
> > Data: Sun, 25 Apr 2004 10:46:30 -0300 (ART)
> >  Assunto: [obm-l] CN 98
> >
> > > Um quadrilátero convexo Q tem diagonais
> > > respectivamente iguais a 4 e 6. Assinale, dentre as
> > > opções, a única possível para o perímetro de Q.
> > >
> > > 10
> > > 15
> > > 20
> > > 25
> > > 30
> > >
> > > _
> >
> > Olá Elton , acho que essa questão foi do ano que eu
> > fiz , 2000 .
> > Use teorema da envoltória e desigualdade triangular.
> > Se as diagonais são x + y = 4  e w + z = 6 e os lados
> > a , b , c e d  , temos :
> > a + b + c + d = 2 ( x + y + z + w )  , [ Desigualdade
> > Triangular]
> > 2p < 20
> >
> > E tb :
> > x + z < a + d + c
> > z + y < a + b + d
> > w + y < a + b + c
> > w + x < b + c + d, Somando tudo :
> > 2( w + x + y + z) < 2 ( a + b + c + d) , [ Teorema da
> > Envoltória]
> > 2p > 10
> >
> > Ou seja  10  < 2p < 20
> >
> > Abraços
> > Luiz H . Barbosa
> > ___
> > __
> > >
> > > Yahoo! Messenger - Fale com seus amigos online.
> > Instale agora!
> > > http://br.download.yahoo.com/messenger/
> > >
> > ===
> > ==
> > > Instruções para entrar na lista, sair da lista e
> > usar a lista em
> > > http://www.mat.puc-rio.br/~nicolau/olimp/obm-l.html
> > >
> > ===
> > ==
> > >
> >
> >
__
> > Acabe com aquelas janelinhas que pulam na sua tela.
> > AntiPop-up UOL - É grátis!
> > http://antipopup.uol.com.br/
> >
> >
> >
> >
=
> > Instruções para entrar na lista, sair da lista e usar a lista em
> > http://www.mat.puc-rio.br/~nicolau/olimp/obm-l.html
> >
=
> >
>
> =
> Instruções para entrar na lista, sair da lista e usar a lista em
> http://www.mat.puc-rio.br/~nicolau/olimp/obm-l.html
> =
>


=
Instruções para entrar na lista, sair da lista e usar a lista em
http://www.mat.puc-rio.br/~nicolau/olimp/obm-l.html
=


[obm-l] Re: [obm-l] A menor bissetriz e o maior lado de um triângulo

2004-04-25 Por tôpico Fellipe Rossi



Bom Rafael, eu tentei por esse 
caminho.
Esteja com lapis e papel para anotar direitinho 
hehehe eh meio grande.
 
Seja ABC o triangulo, o angulo interno de A=2a, 
B=2b e C=2c, onde 2a>2b>2c, logo o maior lado é o lado BC.
 
Agora tome I como incentro de ABC, M o pé da 
bissetriz relativa a BC, N o pé da bissetriz relativa a AC e P o pé da bissetriz 
relativa a AB.
 
Olhando para o triângulo AIB, como a>b podemos 
afirmar que BI>AI
Olhando para o triângulo AIC, como a>c podemos 
afirmar que CI>AI
 
Agora, se conseguirmos provar que IM
 
traçando os raios do círculo inscrito, formamos 3 
triangulos retângulos com I e os pés das bissetrizes.
analisando esses triangulos, podemos dizer, pela 
propriedade dos angulos extermos, que o angulo M vale a+2c e o ângulo N vale 
b+2c.
como 
a+2c > b+2c, 
então MI
Logo, ja sabemos que AM
 
tome o ângulo BPC, ele vale 2a+c.
tome o angulo AMC que vale a+2b  (ambos 
obtidos por ângulo externo)
Como BPC>AMC, o ângulo P do triângulo retângulo 
é menor do que o ângulo M, do outro triângulo retângulo.
Logo IP>MI
 
então, CP>AM
 
Creio que está provado que AM (bissetriz relativa 
ao maior lado) é menos do que BN e CP.
 
O que vocês acham (aqueles que tiveram paciencia de 
ler ate o fim eheh)
 
PS: com um desenho seria bem mais simples 
explicar.
 
Abraços do Rossi
 
 

  - Original Message - 
  From: 
  Johann Peter Gustav Lejeune 
  Dirichlet 
  To: [EMAIL PROTECTED] 
  Sent: Sunday, April 25, 2004 11:41 
  AM
  Subject: Re: [obm-l] A menor bissetriz e 
  o maior lado de um triângulo
  
  Acho que da para ir de trigonometria nao?Depois eu dou uma 
  olhada...rafsanco <[EMAIL PROTECTED]> wrote: 
  Olá 
para todos !Deparei-me com um teorema de geometria euclidiana plana 
que dizia o seguinte: ao maior lado de um triângulo corresponde a 
menor bissetriz. Tentei prová-lo da seguinte forma (infelizmente não 
disponho de recursos visuais, então usem a imaginação ou esboçem o 
desenho num papel para compreenderem melhor o que digo): Seja ABC um 
triângulo qualquer, BC seu maior lado, I seu incentro, x a medida do 
angulo interno de vértice A, y a medida do ângulo interno de vértice B, 
z a medida do ângulo interno de vértice C, AM a bissetriz de x, BO a 
bissetriz de y e CN a bissetriz de z. Sabe-se que x > y e x > 
z uma vez que x é oposto a BC (suposto maior lado). Analisando o 
triângulo AIC, vê-se que x/2 > z/2, logo CI > AI. Observando o 
triângulo AIB é verdadeiro afirmar que x/2 > y/2, portanto BI >! 
AI. Ora IM, IN e IO são segmentos de reta congruentes, visto que são 
raios da circunferência inscrita a ABC, então BI + IO > AI + IM o 
que implica que BO > AM (BI + IO = BO e AI + IM = AM), assim como CI 
+ IN > AI + IM o que implica que CN > AM (CI + IN = CN e AI + IM = 
AM). Enfim, está demonstrada a tese AM < BO e AM < CN. A minha 
demonstração é válida ou há algo nela que a compromete (sei lá, 
algum argumento duvidoso, por exemplo) ? Vocês conhecem alguma outra 
maneira de se provar esse teorema ? Se sim, exponha-a por 
favor.Abraços,Rafael.__Acabe 
com aquelas janelinhas que pulam na sua tela.AntiPop-up UOL - É 
grátis!http://antipopup.uol.com.br/=Instruções 
para entrar na lista, sair da lista e usar a lista 
emhttp://www.mat.puc-rio.br/~nicolau/olimp/obm-l.html=
  
  
  TRANSIRE SVVM PECTVS MVNDOQVE 
  POTIRI
  CONGREGATI EX TOTO ORBE MATHEMATICI OB 
  SCRIPTA INSIGNIA TRIBVERE
  Fields Medal(John Charles 
  Fields)
  
  
  Yahoo! 
  Messenger - Fale com seus amigos online. Instale 
  agora!


[obm-l] Re: [obm-l] A menor bissetriz e o maior lado de um triângulo

2004-04-25 Por tôpico Fellipe Rossi



Rafael, 
Há um erro quando você afirma que IO = IM = IN pois 
são os raios.
Vale lembrar que o raio tem q fazer 90 graus com o 
lado (pois o círculo está inscrito), então essa afirmativa só será válida para 
triângulos equiláteros, ou seja, você particularizou a 
demonstração.
 
Vou dar uma olhada aqui e, caso consiga, mando uma 
proposta de solução.
 
Abraços!
Rossi

  - Original Message - 
  From: 
  Johann Peter Gustav Lejeune 
  Dirichlet 
  To: [EMAIL PROTECTED] 
  Sent: Sunday, April 25, 2004 11:41 
  AM
  Subject: Re: [obm-l] A menor bissetriz e 
  o maior lado de um triângulo
  
  Acho que da para ir de trigonometria nao?Depois eu dou uma 
  olhada...rafsanco <[EMAIL PROTECTED]> wrote: 
  Olá 
para todos !Deparei-me com um teorema de geometria euclidiana plana 
que dizia o seguinte: ao maior lado de um triângulo corresponde a 
menor bissetriz. Tentei prová-lo da seguinte forma (infelizmente não 
disponho de recursos visuais, então usem a imaginação ou esboçem o 
desenho num papel para compreenderem melhor o que digo): Seja ABC um 
triângulo qualquer, BC seu maior lado, I seu incentro, x a medida do 
angulo interno de vértice A, y a medida do ângulo interno de vértice B, 
z a medida do ângulo interno de vértice C, AM a bissetriz de x, BO a 
bissetriz de y e CN a bissetriz de z. Sabe-se que x > y e x > 
z uma vez que x é oposto a BC (suposto maior lado). Analisando o 
triângulo AIC, vê-se que x/2 > z/2, logo CI > AI. Observando o 
triângulo AIB é verdadeiro afirmar que x/2 > y/2, portanto BI >! 
AI. Ora IM, IN e IO são segmentos de reta congruentes, visto que são 
raios da circunferência inscrita a ABC, então BI + IO > AI + IM o 
que implica que BO > AM (BI + IO = BO e AI + IM = AM), assim como CI 
+ IN > AI + IM o que implica que CN > AM (CI + IN = CN e AI + IM = 
AM). Enfim, está demonstrada a tese AM < BO e AM < CN. A minha 
demonstração é válida ou há algo nela que a compromete (sei lá, 
algum argumento duvidoso, por exemplo) ? Vocês conhecem alguma outra 
maneira de se provar esse teorema ? Se sim, exponha-a por 
favor.Abraços,Rafael.__Acabe 
com aquelas janelinhas que pulam na sua tela.AntiPop-up UOL - É 
grátis!http://antipopup.uol.com.br/=Instruções 
para entrar na lista, sair da lista e usar a lista 
emhttp://www.mat.puc-rio.br/~nicolau/olimp/obm-l.html=
  
  
  TRANSIRE SVVM PECTVS MVNDOQVE 
  POTIRI
  CONGREGATI EX TOTO ORBE MATHEMATICI OB 
  SCRIPTA INSIGNIA TRIBVERE
  Fields Medal(John Charles 
  Fields)
  
  
  Yahoo! 
  Messenger - Fale com seus amigos online. Instale 
  agora!


Re: Re:[obm-l] CN 98 [off-topic]

2004-04-25 Por tôpico Fellipe Rossi
O que significa CN?
Pois eu vi uma questão quase idêntica a esta num vestibular da UNICAMP.

Abraços
- Original Message -
From: "rickufrj" <[EMAIL PROTECTED]>
To: "obm-l" <[EMAIL PROTECTED]>
Sent: Sunday, April 25, 2004 12:26 PM
Subject: Re:[obm-l] CN 98


> -- Início da mensagem original ---
>
>   De: [EMAIL PROTECTED]
> Para: [EMAIL PROTECTED]
>   Cc:
> Data: Sun, 25 Apr 2004 10:46:30 -0300 (ART)
>  Assunto: [obm-l] CN 98
>
> > Um quadrilátero convexo Q tem diagonais
> > respectivamente iguais a 4 e 6. Assinale, dentre as
> > opções, a única possível para o perímetro de Q.
> >
> > 10
> > 15
> > 20
> > 25
> > 30
> >
> > _
>
> Olá Elton , acho que essa questão foi do ano que eu
> fiz , 2000 .
> Use teorema da envoltória e desigualdade triangular.
> Se as diagonais são x + y = 4  e w + z = 6 e os lados
> a , b , c e d  , temos :
> a + b + c + d = 2 ( x + y + z + w )  , [ Desigualdade
> Triangular]
> 2p < 20
>
> E tb :
> x + z < a + d + c
> z + y < a + b + d
> w + y < a + b + c
> w + x < b + c + d, Somando tudo :
> 2( w + x + y + z) < 2 ( a + b + c + d) , [ Teorema da
> Envoltória]
> 2p > 10
>
> Ou seja  10  < 2p < 20
>
> Abraços
> Luiz H . Barbosa
> ___
> __
> >
> > Yahoo! Messenger - Fale com seus amigos online.
> Instale agora!
> > http://br.download.yahoo.com/messenger/
> >
> ===
> ==
> > Instruções para entrar na lista, sair da lista e
> usar a lista em
> > http://www.mat.puc-rio.br/~nicolau/olimp/obm-l.html
> >
> ===
> ==
> >
>
> __
> Acabe com aquelas janelinhas que pulam na sua tela.
> AntiPop-up UOL - É grátis!
> http://antipopup.uol.com.br/
>
>
>
> =
> Instruções para entrar na lista, sair da lista e usar a lista em
> http://www.mat.puc-rio.br/~nicolau/olimp/obm-l.html
> =
>
>


=
Instruções para entrar na lista, sair da lista e usar a lista em
http://www.mat.puc-rio.br/~nicolau/olimp/obm-l.html
=


[obm-l] Re: [obm-l] Re:[obm-l] dúvida

2004-04-25 Por tôpico Fellipe Rossi
No caso, não entendi o porque do i*(raiz de 1998), visto que ao elevarmos ao
quadrado,  i^2= -1 e a expressão seria -1998

Creio que apenas (raiz de 1998) seja mais correto.

Porém a questão não deve ser apenas isto, a e b devem pertencer a algum
conjunto específico como os Inteiros...

Abraços,
Rossi

- Original Message -
From: "rickufrj" <[EMAIL PROTECTED]>
To: "obm-l" <[EMAIL PROTECTED]>
Sent: Sunday, April 25, 2004 2:40 AM
Subject: [obm-l] Re:[obm-l] dúvida


> -- Início da mensagem original ---
>
>   De: [EMAIL PROTECTED]
> Para: [EMAIL PROTECTED]
>   Cc:
> Data: Sat, 24 Apr 2004 23:31:13 -0300
>  Assunto: [obm-l] dúvida
>
> > como é que eu resolvo este inequação de maneira
> inteligente!!!
> >
> > | (x+1)/(-x)| >=0
> >
> > a expresão acima está em módulo.
> >
> > outra dúvida é:
> > a x b = 1998 .Sabendo que "a" e "b" são tais números
> que a diferença entre eles seja a menor possível.
> >
> ===
> Na desigualdade ,qualquer valor para x é válido .
> Já na segunda questão , se a e b puderem ser
> complexos , então a = b = i*[sqrt(1998)] .Sendo a
> menor difereça igual a zero.
>
> __
> Acabe com aquelas janelinhas que pulam na sua tela.
> AntiPop-up UOL - É grátis!
> http://antipopup.uol.com.br/
>
>
>
> =
> Instruções para entrar na lista, sair da lista e usar a lista em
> http://www.mat.puc-rio.br/~nicolau/olimp/obm-l.html
> =
>


=
Instruções para entrar na lista, sair da lista e usar a lista em
http://www.mat.puc-rio.br/~nicolau/olimp/obm-l.html
=


[obm-l] Re: Alerta de spam:Re: [obm-l] Re:[obm-l] dúvida

2004-04-25 Por tôpico Fellipe Rossi



Creio que há um erro.
 
Como estamos trabalhando com um módulo, não faz 
sentido analisar os sinais da equação, basta excluirmos o x=0; Visto que não há 
problema algum em a fração resultar em um número negativo pois temos o 
módulo...
 
S={x pertence a R*)
 
Abraços,
Rossi
 
- Original Message - 

  From: 
  [EMAIL PROTECTED] 
  
  To: [EMAIL PROTECTED] 
  Sent: Sunday, April 25, 2004 2:59 
AM
  Subject: Alerta de spam:Re: [obm-l] 
  Re:[obm-l] dúvida
  Qualquer 
  valor para x nao ! Pois x = 0 eh impossivel, porque ele esta no denominador. 
  Talvez a resolucao seria: Primeiramente x <> 0 | 
  (x+1)/(-x)| >=0 | -(x+1)/(x)| >=0 x+1 >=0 x>= 
  - 1 S = {x e R| x >= -1 e x<>0} Em uma 
  mensagem de 25/4/2004 02:43:41 Hora padrão leste da Am. Sul, [EMAIL PROTECTED] escreveu: 
  
  -- Início da mensagem original --- 
 De: [EMAIL PROTECTED] 
   Para: [EMAIL PROTECTED] 
 Cc:    Data: Sat, 24 
Apr 2004 23:31:13 -0300 Assunto: [obm-l] dúvida > como é que 
eu resolvo este inequação de maneira inteligente!!! > > | 
(x+1)/(-x)| >=0 > > a expresão acima está em módulo. 
> > outra dúvida é: > a x b = 1998 .Sabendo que "a" e 
"b" são tais números que a diferença entre eles seja a menor possível. 
> === Na desigualdade ,qualquer valor 
para x é válido . Já na segunda questão , se a e b puderem ser 
complexos , então a = b = i*[sqrt(1998)] .Sendo a menor difereça 
igual a zero. 


[obm-l] Re: [obm-l] Re:Combinatória

2004-04-24 Por tôpico Fellipe Rossi
as questões foram enviadas pelo Gustavo Baggio.

[]'s
- Original Message -
From: "rickufrj" <[EMAIL PROTECTED]>
To: "obm-l" <[EMAIL PROTECTED]>
Sent: Saturday, April 24, 2004 12:44 AM
Subject: [obm-l] Re:Combinatória


> Olá Rossi ,
> (Me desculpe , mandei a mensagem anterior igual a que
> já havia mandado ).
>
> Quanto a questão 2 :
> Na verdade o erro foi meu . A questão é a seguinte :
> Tem que haver no mínimo 2 consoantes entre as vogais .
>
> 5)De quantas maneiras podemos permutar as letras da
> palavra POSTER de tal forma que haja 2 consoantes
> entre as 2 vogais ?
> Obs: Quando ele diz 'haja' , ele está querendo dizer ,
> no mínimo.
> Fiz o problema em 3 partes :
> 1° É a resolução que eu propus inicialmente, com
> somente 2 consoantes entre as vogais .
> 2° Supondo 3 consoantes entre as dua vogais .
> 3°Todas as consoantes entre as duas vogais .
>
> 1°
> Cons = {p,s,t,r}
> Vogais = {o,e}
> Escolha dos lugares das vogais : C(3,1)
> Permutação das vogais : 2!
> Escolha das consoantes : C(4,2)
> Permutação das consoantes : 2!
> Permutação das consoantes que sobraram : 2!
> Portanto a resposta R , será:
> R' = C(3,1) *2! *C(4,2)* 2!*2!
>
> 2°
> Escolha dos lugares das vogais : C(2,1)
> Permutação das vogais : 2!
> Escolha das consoantes : C(4,3)
> Permutação das consoantes : 3!
> Permutação das consoantes que sobraram : 1!
> Portanto a resposta R'' , será:
> R'' = C(2,1* 2! *C(4,3)* 3!* 1!
>
> 3°
> Escolha dos lugares das vogais : C(1,1)
> Permutação das vogais : 2!
> Escolha das consoantes : C(4,4)
> Permutação das consoantes : 4!
> Permutação das consoantes que sobraram : 1!
> Portanto a resposta R''' , será:
> R''' = C(1,1) *2! *C(4,4) *4! *1!
>
> Sendo R a resposta do problema , então :
> R = R'+ R'' + R'''
>
>
>
> Quanto ao problema proposto :
>
> Por exemplo, analise a afirmativa:
> 1) 1 reta possui 1 ponto
> 2) Está certo na minha opinião, visto que mesmo que
> ela tenha infinitos pontos
> não deixa de estar certo dizer que a reta possui um
> ponto.
>
> Na verdade 1 ponto qualquer é igual a outro ponto
> qualquer . Quando vc diz :
> - 1 reta possui o ponto A . Está certo , pois
> subentende-se que esse ponto 'A' é único e diferente
> dos demais .
> Então , quando vc diz :
> - 1 reta possui um ponto .
> E dizem que vc está errado , é porque este único ponto
> é igual aos demais da reta , então na verdade ela não
> possui (1)UM ponto (Notou que esse '1' é a denominação
> do ponto ?) , ela possui infinitos pontos .
>
>
>
> A propósito , mudando de assunto :
> As questões de combinatória foram vc que mandou ?
> É porque eu respondi , mas nem deu pra fazer um
> comentário utilizando o nome do autor , porque eu
> apaguei o e-mail original da  minha caixa antes de
> responder .
>
> Abraços
> Luiz H. Barbosa
>
>
> __
> Acabe com aquelas janelinhas que pulam na sua tela.
> AntiPop-up UOL - É grátis!
> http://antipopup.uol.com.br/
>
>
>
> =
> Instruções para entrar na lista, sair da lista e usar a lista em
> http://www.mat.puc-rio.br/~nicolau/olimp/obm-l.html
> =
>


=
Instruções para entrar na lista, sair da lista e usar a lista em
http://www.mat.puc-rio.br/~nicolau/olimp/obm-l.html
=


[obm-l] Re: [obm-l] Re:Combinatória

2004-04-23 Por tôpico Fellipe Rossi
Luis, discordo com a sua solução da questão do PÔSTER.
No caso você fez que APENAS 2 vogais estão entre as consoantes. Porém eu
interpretei que o enunciado pedia 2 ou +, visto que havendo 3 ou 4 vogais
entre as consoantes não deixamos de ter 2. Eu mandei minha proposta de
resolução hoje cedo, caso queira ver a minha interpretação.

Por exemplo, analise a afirmativa:
1) 1 reta possui 1 ponto
Está certo na minha opinião, visto que mesmo que ela tenha infinitos pontos
não deixa de estar certo dizer que a reta possui um ponto.

Eu acho que o enunciado está mal formulado ele deveria ter usado palavras do
tipo APENAS ou PELO MENOS, a não ser que a parte de interpretação seja uma
"pegadinha". Gostaria de uma terceira opinião.

Abraços,
Rossi

- Original Message -
From: "rickufrj" <[EMAIL PROTECTED]>
To: "obm-l" <[EMAIL PROTECTED]>
Sent: Friday, April 23, 2004 5:01 PM
Subject: [obm-l] Re:Combinatória


> 1)Dentre todos os números de 7 dígitos , quantos
> possuem exatamente 3 dígitos  9 e os 4 dígitos
> restantes todos diferentes ?
>
> Suas opções de dígitos estão no conjunto I =
> {0,1,...,8}
> Escolhendo os 3 lugares para os 9's , temos  C (7,3) .
> Agora complentando o restante dos dígitos do número
> com as opções de I ,e sendo o número de opções P,
> temos :
> P = 9*8*7*6
> Mas espere , devemos retirar os números que começam
> com ZERO .Com isso I' = { 1,2,...,8} :
> Fixando o ZERO no 1° dígito , temos :
> Escolhendo onde os 9's ,temos  C(6,3) .
> Complentando o restante dos dígitos do número com
> opções de I' ,e sendo o número de opções P', temos :
> P' = 8*7*6
> Portanto a resposta R , será:
> R = [C (7,3) *P]  -  [C(6,3) *P']
>
> 2)No sistema decimal ,quantos números de 6 dígitos
> distintos possuem 3 dígitos pares e 3 dígitos ímpares ?
>
> Cp = { 0,2,4,6,8}  e Ci = { 1,3,5,7,9}
> Escolhendo os ímpares dentro de Ci e sendo o número de
> opções O , temos :
> O = 5*4*3
> Escolhendo os lugares onde estes 3 ímpares ficaram ,
> temos :
> C(6,3)
> Agrupando os pares nos locais restantes e sendo P o
> número de opções para se fazer isto , temos :
> P = 5*4*3
> Mas espere , como no exercício 1 , deveremos tirar os
> casos em que o ZERO esta no primeiro dígito :
> Fixando o ZERO no 1° dígito :
> Escolhendo os ímpares dentro de Ci e sendo o número de
> opções O' , temos:
> O' = 5*4*3
> Escolhendo os lugares onde estes 3 ímpares ficaram ,
> temos :
> C(5,3)
> E finalmente agrupando os pares de Cp' = {2,4,6,8} nos
> locais restantes e sendo P' o número de opções para se
> fazer isto , temos :
> P' = 4*3
> Portanto a resposta R , será:
> R = [O* C(6,3)*5*4*3] - [O'*C(5,3)* P']
>
> 3)Dentre as permutações dos 10 dígitos (0,1,...,9)
> quantas são aquelas em que o primeiro dígito é maior
> do que 1 e o último dígito é menor do que 7 ?
>
> Dividindo em casos , temos :
> 1°Caso  1 < (1°dig.) < 7  e   1 >= (10° dig.) >= 0
> 2°Caso  7 <= (1° dig.) <= 9  e  7 > (10° dig.) > 1
> Faça os casos separadamente , e some os resultados !
>
> 5)De quantas maneiras podemos permutar as letras da
> palavra POSTER de tal forma que haja 2 consoantes
> entre as 2 vogais ?
> Cons = {p,s,t,r}
> Vogais = {o,e}
> Escolha dos lugares das vogais : C(3,1)
> Permutação das vogais : 2!
> Escolha das consoantes : C(4,2)
> Permutação das consoantes : 2!
> Permutação das consoantes que sobraram : 2!
> Portanto a resposta R , será:
> R = C(3,1) *2! *C(4,2)* 2!*2!
>
> O número 4 eu fiz de uma forma que eu abri em vários
> casos , como achei que ficou muito grande , alguem
> deve mandar uma solução mais simples .
> Espero ter ajudado .. : )
>
> Abraços
> Luiz H. Barbosa
>
>
>
>
>
>
>
>
>
>
>
>
>
>
>
>
>
>
>
> __
> Acabe com aquelas janelinhas que pulam na sua tela.
> AntiPop-up UOL - É grátis!
> http://antipopup.uol.com.br/
>
>
>
> =
> Instruções para entrar na lista, sair da lista e usar a lista em
> http://www.mat.puc-rio.br/~nicolau/olimp/obm-l.html
> =
>
>


=
Instruções para entrar na lista, sair da lista e usar a lista em
http://www.mat.puc-rio.br/~nicolau/olimp/obm-l.html
=


[obm-l] combinatória LXV

2004-04-23 Por tôpico Fellipe Rossi



Ops, me esqueci de um exercício
Essa numeração me atrapalhou um pouco 
hehe
 
lá vai:
 
Analisaremos 3 vertentes nesse 
exercício:
 
i) começar por maior q 1 e terminar por menor que 
1
ii) começar por maior que 6 e terminar por 
2<=x<=6
iii) começar por 2<=x<=6 e terminar por 
2<=x<=6.
 
i) 1o. 8, último: 2, intermediário: 8! 
=>
=> 8*2*8! = 645.120
ii) 1o.: 3, último: 5, intermediários: 8! 
=>
=> 3*5*8! = 604.800
iii) 1o.: 5, último: 4, intermediários: 8! 
=>
=> 5*4*8! = 806.400
 
Total: 2.056.320 números.
 
Espero ter ajudado! qualquer dúvida: [EMAIL PROTECTED]
Rossi


[obm-l] dúvida sobre a lista.

2004-04-23 Por tôpico Fellipe Rossi



Pessoal sou novo na lista e tenho notado que minhas 
mensagens demoram muito para chegar.
Eu digo isos porque participo de outras listas e as 
mensagens são quase que instantâneas.
Gostaria de saber se isso é normal.
 
Abraços do Rossi


[obm-l] Re: [obm-l] combinatória LXV

2004-04-23 Por tôpico Fellipe Rossi



1)
teremos 999_ _ _ _
ou seja, será uma permutação do tipo AAABCDE (com 
repetição)
para evitarmos de contar mais de uma vez cada 
número, iremos primeiro escolher os 4 numeros e depois permutaremos. para 
escolhe-los, consideramos que a ordem nao importa. Entao:
-> C9,4
Agora permutaremos os 7 elementos (com repetição de 
3) 
-> P7,3
Total Parcial = C9,4 * P7,3 = 
105.840
 
Porém devemos desconsiderar os numeros começados 
por 0. 
0 _ _ _ _ _ _
Seria então C8,3 * P6,3 = 6.720
 
Total = 105.840 - 6.720 = 99.120 
números.
 
 
 
2)
Existem 2 maneiras de resolver este 
problema:
 
i) P P P I I I 

Vamos permutar para verificar todas as posições 
disponíveis.
P6,3,3 (repeticao de 3 e 3) = 20.
Agora para cada posição teremos: 5*4*3(pares) 
*5*4*3 (impares) =>
=> 20*60*60 = 72.000
 
Porém precisamos descontar aqueles que começam por 
0
0 P P I I I
Faremos o mesmo processo.
P5,3,2 = 10.
Para cada posição teremos: 4*3 (pares) *5*4*3 
(impares) =>
=> 10*12*60 = 7.200
Logo, total = 72.000 - 7.200 = 64.800 
números.
 
ii) P P P I I I
Nesta outra maneira, priemiramente escolheremos os 
números e depois permutaremos.
C5,3 (pares) C5,3 (impares)
Agora permutaremos os 6:  P6
isso dá: 10x10x720 = 72.000
Agora excluiremos os que começam por 
0.
 
0 _ _ _ _ _
Escolhendo: C4,2 (pares) e C5,3 
(impares)
E permutaremos os 5: P5
isso dá: 6*10*120 = 7.200
 
Logo o total será 72.000 - 7.200 = 
64.800
 
 
3) esta nao é para ser feita!?
 
4) neste caso, ao meu ver, o melhor a se fazer 
é Total - Negação; visto que o enunciado nao quer APENAS 2 
consoantes.
ou seja, veremos as possibilidades em que O e E 
estao  juntos e as em q apenas 1 casa os separam, e subtraimos do total 
pois acharemos as com 2, 3 e 4 consoantes os separando.
 
i) OE _ _ _ _  temos 5 possibilidades de OE 
juntos, então teremos 4! (ordem das consoantes) * 2! (troca de O e E entre si) * 
5 (total de formas q O e E estao juntos)
isso dá: 24 * 2 * 5 = 240 
possibilidades.
 
ii) O _ E _ _ _ temos 4 formas deste tipo de 
comportamento, então teremos 4! * 2! * 4 = 192 possibilidades
 
Bom, total = 6! 
Então o q nos serve será 6! - 240 - 192 = 720 - 438 
= 292 formas diferentes.
 
 
Abraços,
Fellipe Rossi

  - Original Message - 
  From: 
  Gustavo 
  Baggio 
  To: [EMAIL PROTECTED] 
  Sent: Friday, April 23, 2004 12:57 
  AM
  Subject: [obm-l] combinatória LXV
  
  Valeu Felipe, valeu Augusto...
   
  Tem mais exercícios que to me encrecando, ainda mais que eu invoquei de 
  fazer todos que tem aqui (92), se alguém pudesse me ajudar (again, again, 
  again  )
   
  >> Dentre todos os números de 7 dígitos, quantos possuem exatamente 
  3 dígitos 9 e os 4 dígitos restantes todos diferentes?
   
  1 No sistema decimal, quantos números de 6 dígitos distintos possuem 
  3 dígitos pares e 3 dígitos ímpares?
   
  2 Dentre as permutaçães dos 10 dígitos (0, 1, 2, ... , 9) quantas 
  são aquelas em que o primeiro digito é maior do que 1 e o último digitos é 
  menor do que 7?
   
  3 Um bote tem 8 lugares, 4 frente e 4 atras. De quantas maneiras 
  podemos escolher um tripulação para o bote se dos 31 candidatos, 10 preferem 
  frente, 12 preferem atras e 9 não tem preferência.
   
  4 De quantas maneiras podemos permutar as letras da palavra PÔSTER 
  de tal forma que haja 2 consoantes entre as 2 vogais?
   
  Respostas.
  4. 3 x 48
  1.64.800
  2. 2.056.320
  Gustavo
  ===
  
  
  Yahoo! 
  Messenger - Fale com seus amigos online. Instale 
  agora!


[obm-l] Re: [obm-l] errata combinatória

2004-04-23 Por tôpico Fellipe Rossi



o 4o. ou o numero 4?

  - Original Message - 
  From: 
  Gustavo 
  Baggio 
  To: [EMAIL PROTECTED] 
  Sent: Friday, April 23, 2004 2:20 
AM
  Subject: [obm-l] errata 
combinatória
  
  esqueçam o 4to... que passei
  
  
  Yahoo! 
  Messenger - Fale com seus amigos online. Instale 
  agora!


Re: [obm-l] combinatoria ......

2004-04-21 Por tôpico Fellipe Rossi



1) 
Vamos considerar 5 grupos de 2 cadeiras (cada grupo 
para 1 casal) e 5 cadeiras vazias.
Vamos permuta-los de forma q tenhamos todas as 
posições possiveis e então trocar cada marido de posicao com sua mulher (tipo, M 
à esquerda e marido a direita, trocando).
P10,5(repeticao de 5 = vazios) *2^5 = 10!*32/5! = 
967680
 
Note que não consideramos os pares q serão ocupados 
pelos casais como repetições visto que cada casal eh diferente do 
outro.
 
 
2) obs1: a ordem importa.
    obs2: 13 cartas de cada 
naipe.
 
Temos 2 vertentes. 
i) começando com o rei ou o valete de 
ouros.
ii) começando por uma carta de ouros qualquer (sem 
ser K e J)
 
i.a) J*, K, J = 1*4*3 = 12
i.b) K*, K, J = 1*3*4 = 12
 
ii) 11 * 4 * 4 = 176
 
Logo no total são 200 possibilidades.
 
Espero ter ajudado,
Rossi

  - Original Message - 
  From: 
  Gustavo 
  Baggio 
  To: [EMAIL PROTECTED] 
  Sent: Wednesday, April 21, 2004 11:37 
  PM
  Subject: [obm-l] combinatoria 
..
  
  >> Há 15 cadeiras em uma fila. De quantos modos 5 casais podem se 
  sentar nas cadeiras se nenhum marido senta separado de sua mulher.
   
  >> De um baralho comum (52 cartas) retiram-se sucessivamente e sem 
  reposição 3 cartas. Quantas são as extrações nas quais a primeira carta é de 
  ouros; e segunda é um rei e a terceira é um valete?
   
  Gustavo..._Gustavo 
  Baggio [EMAIL PROTECTED] iCQ UIN 63522252"Why use 
  WINDOWS if you can use doors?" - Be free, use LINUX!Powered by Debian 
  Linux
  
  
  Yahoo! 
  Messenger - Fale com seus amigos online. Instale 
  agora!


Re: [obm-l] Geometria Plana

2004-04-21 Por tôpico Fellipe Rossi



Bom, considerando que FG seja sobre o lado 
BC:
 
Observe que por se tratar de um retângulo, ED // 
BC
Logo os triangulos AED e ABC são 
semelhantes.
 

Chamaremos EF=DG=x e consequentemente, 
FG=ED=3x
 
A altura de AED mede 8-x e a base, 
3x
 
entao: 8-x/8 = 3x/12 => x=8/3
 
o perímetro é 8x = 8*8/3 = 64/3.
 
Abraços, Rossi

  - Original Message - 
  From: 
  Fabio Contreiras 
  To: [EMAIL PROTECTED] 
  Sent: Wednesday, April 21, 2004 10:32 
  PM
  Subject: Re: [obm-l] Geometria 
Plana
  
  FG é a base
  EF altura
   
  foi mal hehe, eh pq  a figura ta em 
  arquivo.
  
- Original Message - 
From: 
    Fellipe Rossi 
To: [EMAIL PROTECTED] 
Sent: Wednesday, April 21, 2004 9:51 
PM
Subject: Re: [obm-l] Geometria 
Plana

creio que saber sobre qual lado estarão 2 
pontos do retângulo faz a diferença.
tente descrever a figura :)
 
Abraços,
Rossi

  - Original Message - 
  From: 
  Fabio Contreiras 
  To: [EMAIL PROTECTED] 
  Sent: Wednesday, April 21, 2004 8:59 
  PM
  Subject: Re: [obm-l] Geometria 
  Plana
  
  Opa, eh so isso mesmo.. desconsidere akele " 
  NA FIGURA"
  eh pq no exercicio vem uma figura... mas eh 
  facil de visualizar... um retangulo inscrito num triangulo e 
  talz...
  abraços!
  
- Original Message - 
From: 
Alan Pellejero 
To: [EMAIL PROTECTED] 
Sent: Wednesday, April 21, 2004 
8:26 PM
Subject: Re: [obm-l] Geometria 
Plana

Acho que você não terminou de escrever o exercício...
Fabio Contreiras <[EMAIL PROTECTED]> 
wrote:

  
  

  Ae , se alguem puder me dar uma help 
  ae!
   
  Abraços!
   
   
  1 ) Lado BC de um triangulo ABC mede 
  12cm, e a altura relativa ao lado BC mede 8 cm. Se FG = 3 EF , então o 
  perimetro do RETANGULO inscrito no triangulo DEFG, em cm, é 
  ?
   
   
  NA FIGURA


Yahoo! 
Messenger - Fale com seus amigos online. Instale 
agora! 


Esta mensagem foi verificada 
pelo E-mail Protegido 
Terra.Scan engine: VirusScan / Atualizado em 21/04/2004 / 
Versão: 1.5.2Proteja o seu e-mail Terra: http://www.emailprotegido.terra.com.br/ 



E-mail classificado pelo Identificador de Spam Inteligente.Para 
alterar a categoria classificada, visite http://www.terra.com.br/centralunificada/emailprotegido/imail/imail.cgi?+_u=fabiocontreiras&_l=1082591615.945523.15392.pamplona.terra.com.br 
  


Esta mensagem foi verificada pelo E-mail Protegido 
Terra.Scan engine: VirusScan / Atualizado em 21/04/2004 / Versão: 
1.5.2Proteja o seu e-mail Terra: http://www.emailprotegido.terra.com.br/ 



E-mail classificado pelo Identificador de Spam Inteligente.Para alterar 
a categoria classificada, visite http://www.terra.com.br/centralunificada/emailprotegido/imail/imail.cgi?+_u=fabiocontreiras&_l=1082596790.843810.29452.laranjal.terra.com.br 
  


Re: [obm-l] Geometria Plana

2004-04-21 Por tôpico Fellipe Rossi



Entao, FG está sobre o lado BC, certo? 
:)

  - Original Message - 
  From: 
  Fabio Contreiras 
  To: [EMAIL PROTECTED] 
  Sent: Wednesday, April 21, 2004 10:32 
  PM
  Subject: Re: [obm-l] Geometria 
Plana
  
  FG é a base
  EF altura
   
  foi mal hehe, eh pq  a figura ta em 
  arquivo.
  
- Original Message - 
From: 
Fellipe Rossi 
To: [EMAIL PROTECTED] 
Sent: Wednesday, April 21, 2004 9:51 
PM
Subject: Re: [obm-l] Geometria 
Plana

creio que saber sobre qual lado estarão 2 
pontos do retângulo faz a diferença.
tente descrever a figura :)
 
Abraços,
Rossi

  - Original Message - 
  From: 
  Fabio Contreiras 
  To: [EMAIL PROTECTED] 
  Sent: Wednesday, April 21, 2004 8:59 
  PM
  Subject: Re: [obm-l] Geometria 
  Plana
  
  Opa, eh so isso mesmo.. desconsidere akele " 
  NA FIGURA"
  eh pq no exercicio vem uma figura... mas eh 
  facil de visualizar... um retangulo inscrito num triangulo e 
  talz...
  abraços!
  
- Original Message - 
From: 
Alan Pellejero 
To: [EMAIL PROTECTED] 
Sent: Wednesday, April 21, 2004 
8:26 PM
Subject: Re: [obm-l] Geometria 
Plana

Acho que você não terminou de escrever o exercício...
Fabio Contreiras <[EMAIL PROTECTED]> 
wrote:

  
  

  Ae , se alguem puder me dar uma help 
  ae!
   
  Abraços!
   
   
  1 ) Lado BC de um triangulo ABC mede 
  12cm, e a altura relativa ao lado BC mede 8 cm. Se FG = 3 EF , então o 
  perimetro do RETANGULO inscrito no triangulo DEFG, em cm, é 
  ?
   
   
  NA FIGURA


Yahoo! 
Messenger - Fale com seus amigos online. Instale 
agora! 


Esta mensagem foi verificada 
pelo E-mail Protegido 
Terra.Scan engine: VirusScan / Atualizado em 21/04/2004 / 
Versão: 1.5.2Proteja o seu e-mail Terra: http://www.emailprotegido.terra.com.br/ 



E-mail classificado pelo Identificador de Spam Inteligente.Para 
alterar a categoria classificada, visite http://www.terra.com.br/centralunificada/emailprotegido/imail/imail.cgi?+_u=fabiocontreiras&_l=1082591615.945523.15392.pamplona.terra.com.br 
  


Esta mensagem foi verificada pelo E-mail Protegido 
Terra.Scan engine: VirusScan / Atualizado em 21/04/2004 / Versão: 
1.5.2Proteja o seu e-mail Terra: http://www.emailprotegido.terra.com.br/ 



E-mail classificado pelo Identificador de Spam Inteligente.Para alterar 
a categoria classificada, visite http://www.terra.com.br/centralunificada/emailprotegido/imail/imail.cgi?+_u=fabiocontreiras&_l=1082596790.843810.29452.laranjal.terra.com.br 
  


Re: [obm-l]

2004-04-21 Por tôpico Fellipe Rossi



Bem pensado! hehe
minha resolução pediu mais calculos, apesar de tb 
ser simples
 
só uma obs: faltou dizer que a hipotenusa será 
formada por 3-r + 4-r devido a propriedade de tangentes partindo do mesmo ponto. 
:)
 
Abraços

  - Original Message - 
  From: 
  Renato 
  Cubellas de Azevedo 
  To: [EMAIL PROTECTED] 
  Sent: Wednesday, April 21, 2004 10:23 
  PM
  Subject: Re: [obm-l] 
  
  3x+4y=12
  Dividindo por 12:
  x/4+y/3=1
  A reta da equação corta os eixos nos pontos (4,0) e 
  (0,3).
   
  Como a circunferencia esta inscrita no triangulo, 
  fica facil deduzir que as Coordenadas do centro é (r,r) 
  (r=raio).
   
  A reta forma com os eixos um triangulo pitagórico (com 
  hipotenusa =5), e desenhando a figura, chega-se facilmente a conclusão de que 
  a hipotenusa = 3-r+4-r
   
  entaum, sendo 3-r+4-r=5, r=1
   
  então a equação da circunferencia é: 
   
  (x-r)²+(y-r)²=r²
  (x-1)²+(y-1)²=1
  
- Original Message - 
From: 
Maurizio 

To: [EMAIL PROTECTED] 
Sent: Wednesday, April 21, 2004 8:25 
PM
Subject: [obm-l] 

Tou querendo fazer esse 
problema:Obtenha a eq da circunferencia inscrita no triangulo 
formado pela reta 3x+4y=12 e os eixos x e y.Obrigado 

  
 


Re: [obm-l] Geometria Plana

2004-04-21 Por tôpico Fellipe Rossi



creio que saber sobre qual lado estarão 2 pontos do 
retângulo faz a diferença.
tente descrever a figura :)
 
Abraços,
Rossi

  - Original Message - 
  From: 
  Fabio Contreiras 
  To: [EMAIL PROTECTED] 
  Sent: Wednesday, April 21, 2004 8:59 
  PM
  Subject: Re: [obm-l] Geometria 
Plana
  
  Opa, eh so isso mesmo.. desconsidere akele " NA 
  FIGURA"
  eh pq no exercicio vem uma figura... mas eh facil 
  de visualizar... um retangulo inscrito num triangulo e talz...
  abraços!
  
- Original Message - 
From: 
Alan Pellejero 
To: [EMAIL PROTECTED] 
Sent: Wednesday, April 21, 2004 8:26 
PM
Subject: Re: [obm-l] Geometria 
Plana

Acho que você não terminou de escrever o exercício...
Fabio Contreiras <[EMAIL PROTECTED]> 
wrote:

  
  

  Ae , se alguem puder me dar uma help 
  ae!
   
  Abraços!
   
   
  1 ) Lado BC de um triangulo ABC mede 12cm, e 
  a altura relativa ao lado BC mede 8 cm. Se FG = 3 EF , então o perimetro 
  do RETANGULO inscrito no triangulo DEFG, em cm, é ?
   
   
  NA FIGURA


Yahoo! 
Messenger - Fale com seus amigos online. Instale 
agora! 


Esta mensagem foi verificada pelo E-mail Protegido 
Terra.Scan engine: VirusScan / Atualizado em 21/04/2004 / Versão: 
1.5.2Proteja o seu e-mail Terra: http://www.emailprotegido.terra.com.br/ 



E-mail classificado pelo Identificador de Spam Inteligente.Para alterar 
a categoria classificada, visite http://www.terra.com.br/centralunificada/emailprotegido/imail/imail.cgi?+_u=fabiocontreiras&_l=1082591615.945523.15392.pamplona.terra.com.br 
  


Re: [obm-l]

2004-04-21 Por tôpico Fellipe Rossi



Colocando a reta no par de eixos coordenados, 
chamaremos de A o ponto onde ela corta o eixo x (4) e B onde corta o eixo y 
(3).
 
O centro circunferência inscrita é o encontro 
das bissetrizes, oq faremos é achar as equações das retas q representam as 
bissetrizes e igualá-las para achar o centro.
 
pegando a bissetriz do vértice O, temos 
y=x.
pegando o vértice B, acharemos a tangente da metade 
do ângulo OBA (chamaremos OBA de 2i) para determinar a reta. 

Sabemos que tg(2i)=3/4
 
ou seja, tg (i+i)=3/4 
=> tg(i)+tg(i) / 1- tg(i)*tg(i)=3/4
 
=> resolvendo báskara acharemos que tg(i)=1/3 ou 
tg(i)= -3. Como o ângulo i é agudo, consideraremos apenas 
tg(i)=1/3.
 
agora, sendo y=ax+b, temos que tg(i)=b/4 

=> b=4/3
 
a=tg(180º-i) = -tg(i) = -1/3.
 
logo a equação da reta será: 
y=-1/3x+4/3
igualando...
x=-1/3x+4/3 => x=1;
 
Logo o centro será o ponto (1,1). Como 
a circunferência tangencia tanto o eixo x quanto o eixo y, o raio também 
será 1.
 
Então a eq da reta é:  (x-1)^2 + (y-1)^2 = 
1
 
Espero ter ajudado,
Rossi
 
PS: a explicação ficaria bem menor se pudesse por 
um desenho :)
 

  - Original Message - 
  From: 
  Maurizio 
  
  To: [EMAIL PROTECTED] 
  Sent: Wednesday, April 21, 2004 8:25 
  PM
  Subject: [obm-l] 
  Tou querendo 
  fazer esse problema:Obtenha a eq da circunferencia inscrita no 
  triangulo formado pela reta 3x+4y=12 e os eixos x e 
  y.Obrigado 


Re: [obm-l] GEOMETRIA

2004-04-21 Por tôpico Fellipe Rossi



Pessoal, sou novo na lista e não sei até que ponto 
posso estar postando anexos.
No caso desta questão é impossível descrever a 
resolução sem uma figura, então eu publiquei em um website. Gostaria de saber se 
da próxima vez posso postar como anexo.
 
o endereço é:
 
http://www.questoes.bravenet.com/questao.JPG
 
Abraços, 
Rossi

  - Original Message - 
  From: 
  Andre 
  To: [EMAIL PROTECTED] 
  Sent: Wednesday, April 21, 2004 12:28 
  AM
  Subject: [obm-l] GEOMETRIA
  
  
  Num triângulo ABC, retângulo em A e isósceles, 
  seja D um ponto do lado AC (D diferente de A e C) e seja E o ponto do 
  prolongamento do lado BA tal que o triângulo ADE é isósceles. Se P é o ponto 
  médio do segmento BD, R é o ponto médio do segmento CE e Q o ponto onde se 
  cortam as retas ED e BC, demonstre  que o quadrilátero ARQP é um 
  quadrado.


Re: [obm-l] Eureka_18

2004-04-21 Por tôpico Fellipe Rossi
para concluir que f (m)=m, basta fazer:

seja m+f(n) = 2003.
então f(n) = 2003 - m
=> f(2003) = f(f(m)) + f(n)
=> 2003 = f(f(m)) + 2003 - m
=> m = f(f(m))

Porém ainda estou tentando terminar.

Abraços,
Rossi

- Original Message -
From: "Ricardo Bittencourt" <[EMAIL PROTECTED]>
To: <[EMAIL PROTECTED]>
Sent: Tuesday, April 20, 2004 3:07 PM
Subject: Re: [obm-l] Eureka_18


> rickufrj wrote:
>
> > 83) Seja N = {0,1,2,3, ..}
> > Determine quantas funções   satisfazem f(2003) = 2003,
> > f(n) <= 2003 para todo n <= 2003 e f(m + f(n)) = f(f
> > (m)) + f(n) , para todo m,n pertence N.
> >
> > Estou tentando resolve-lo e gostaria da ajuda de
> > vocês .
> > O que eu fiz :
> > f(m + f(n)) = f(f(m)) + f(n)
> > f(m + f(n)) = m + f(n)
>
> Como você concluiu que f(f(m))=m nessa passagem?
> Pra mim isso aqui tá bizarro.
>
> O que daria pra fazer é
> f(m+f(n))=f(f(m))+f(n)
> f(0+f(0))=f(f(0))+f(0)
> f(f(0))=f(f(0))+f(0)
> f(0)=0
>
> f(m+f(n))=f(f(m))+f(n)
> f(0+f(n))=f(f(0))+f(n)
> f(f(n))=f(n)
>
> Mas f(f(n))=f(n) não implica necessariamente em f(n)=n,
> por exemplo, pegue f(n) como sendo "a menor potência de dois
> menor ou igual a n". Nesse caso:
>
> f(f(1))=f(1) pois f(2)=2
> f(f(3))=f(3) pois f(4)=4
> f(f(7))=f(7) pois f(8)=8
>
> mas
>
> f(1)=2
> f(3)=4
> f(7)=8
>
> 
> Ricardo Bittencourt   http://www.mundobizarro.tk
> [EMAIL PROTECTED]   "tenki ga ii kara sanpo shimashou"
> -- União contra o forward - crie suas proprias piadas --
> =
> Instruções para entrar na lista, sair da lista e usar a lista em
> http://www.mat.puc-rio.br/~nicolau/olimp/obm-l.html
> =
>
>


=
Instruções para entrar na lista, sair da lista e usar a lista em
http://www.mat.puc-rio.br/~nicolau/olimp/obm-l.html
=